SlideShare una empresa de Scribd logo
1 de 132
NEUROLOGIA
ENARM
MANIFESTACIONES DE LESION AL SNC
• Como se muestra una lesión del TRACTO ÓPTICO?
• R = Como CUADRANTOPSIA homónima contralateral.
• Que caracteriza al síndrome de Horner?
• R = Miosis, ptosis y anhidrosis debido a lesión de las vías del SISTEMA SIMPÁTICO ispolaterales.
1) Miosis: Se debe a denervación simpática del musculo tarsal
2) Anhidrosis: Consecuencia de la denervación simpática del musculo tarsal
3) Para localizar la lesión se realizan pruebas con cocaína, hidroxifentamina y fenilefrina.
• Que ocasiona las pupilas de Argyll Robertson?
1) Son PUPILAS IRREGULARES Y MÁS PEQUEÑAS de lo normal en la oscuridad.
2) Demuestran falta del reflejo fotomotor, tanto directo como consensual, sin embargo conservan la
acomodación.
3) Se asocia a NEUROSIFILIS.
• Que ocasiona las pupilas de Holmes-Adie?
1) Se trata de una PUPILA GRANDE Y TÓNICA de forma irregular
2) Tiene reacción pobre ante estimulo luminoso
3) Suele indicar una lesión de las vías parasimpáticas posganglionares, a nivel del ganglio ciliar o los nervios
ciliares cortos.
4) Se ve una REACCIÓN CONSTRICTORA EXAGERADA ante la administración de agonistas muscarinicos como la
meticolina o la PILOCARPINA.
5) Se asocian a reflejos de estiramiento muscular disminuidos o ausentes.
MANIFESTACIONES DE LESION AL SNC
• Cuales son las manifestaciones oculares en caso de obstrucción de las RADIACIONES
ÓPTICAS?
• R = CUADRANTOPSIAS HOMONIMAS
• -A 75-year-old woman presents with symptoms of visual change and facial weakness. On
examination, the pupils are equal and reactive to light, the fundi appear normal, and there is
a right homonymous visual field defect. Which of the following is the most likely cause of the
right homonymous hemianopia?
• (A) right optic nerve
• (B) chiasm
• (C) right optic radiations
• (D) right occipital lobe
• (E) left optic radiations
• - The hemianopia is due to a lesion of the left optic radiations. The posterior cerebral artery
arises from the basilar artery but is sometimes a branch of the internal carotid. With
posterior cerebral artery lesions affecting the occipital cortex, it is possible for the
hemianopia to be an isolated finding.
• CORRELACIÓN TOPOGRÁFICA DE DEFECTOS DEL CAMPO VISUAL. A: abolición CV izquierdo (lesión nervio óptico izquierdo). B: escotoma
yuncional. C: hemianopsia bitemporal heterónima (lesión quiasmática). D: hemianopsia homónima derecha (lesión quiasmática). E:
sectoronopsia cuádruple homónima derecha (lesión ganglio geniculado izquierdo). F: Sectoranopsia horizontal derecha (lesión ganglio
geniculado izquierdo). G: Cuadrantanopsia homónima inferior derecha (lesión lóbulo parietal izquierdo). H: Cuadrantanopsia homónima
superior derecha (lesión lóbulo temporal izquierdo). I: Hemianopsia homónima derecha con respeto macular (lesión occipital izquierda).
MANIFESTACIONES DE LESION AL SNC
• Que es la disautonomia?
• R = Proceso patológico central o periférico que se manifiesta por anomalías de la regulación
de la TA, diaforesis termorreguladora, función sexual.
• Que es el mioclono?
1) Es común en epilepsia, presentando ESPASMOS. Se presenta en casos de anoxia,
hiperuricemia.
2) El tratamiento es con levodopa, privación alcohólica y responde a acido valproico y
benzodiacepinas.
• Que motoneurona se ve afectada en la espasticidad?
• R = MOTONEURONA SUPERIOR
• Cual es tratamiento de espasticidad?
• R = Fisioterapia, dantroleno debilita la contractura muscular al interferir con el calcio,
diazepam, inyección de fenol para disminuir la espasticidad selectiva o toxina botulínica.
MANIFESTACIONES DE LESION AL SNC
• En que zona se encuentra la lesión en un paciente estuporoso/comatoso en ausencia
unilateral de respuesta a pesar de la aplicación de estímulos en ambos lados del cuerpo?
• R = CORTICOESPINAL
• En que zona se encuentra la lesión en paciente en ausencia bilateral de respuesta a
estímulos?
• R = Afección de TALLO ENCEFÁLICO, lesión bilateral de vía piramidal
• La postura de decorticacion a que daño se debe?
• R = Lesión de CAPSULA INTERNA y del PEDÚNCULO CEREBRAL ANTERIOR.
• La postura de decerebracion a que daño se debe?
• R = DESTRUCCIÓN MESENCÉFALO y de la porción anterior del puente de vacilio
• Que causa el síndrome de encerramiento o estado desaferentado?
1) Lesiones agudas que afectan principalmente la PARTE ANTERIOR DEL PUENTE DE VACILIO Y
RESPETAN EL TEGUMENTO.
2) Tienen respuesta ocular y están consientes pero en estado comatoso y se RECUPERAN DE
2-3ª.
Posición Decorticación Posición Descerebración
MOTONEURONAS
• Que caracteriza a las lesiones de la motoneurona?
• R = Debilidad sin perdida sensorial
• Cual es la clasificación de la enfermedad de motoneurona y que caracteriza a cada una de ellas?
1) Parálisis bulbar progresiva: degeneración que predomina en núcleos motores de pares
craneales
2) Parálisis pseudobulbar: se debe a enfermedad cortico bulbar alterando neurona motora
superior
3) Atrofia muscular espinal progresiva: Hay degeneración de las astas anteriores de la medula
espinal afectando a neurona motora inferior
4) Esclerosis lateral primaria: Déficit puro de neurona motora superior en extremidades inferiores
5) Esclerosis lateral aminotrofica: Déficit mixto de NMS e INFERIOR en extremidades pudiendo
alterar el estado cognitivo con DEMENCIA FRONTOTEMPORAL O PARKINSONISMO.
• Cual es el cuadro clínico de enfermedad degenerativa de neuronas motoras?
1) Alteración bulbar: dificultad para la deglución, masticación, tos, respiración y disartria.
2) Parálisis bulbar progresiva: descanso del paladar disminuyendo reflejo nauseoso, tos débil y
desgaste muscular. Lingual con fasciculaciones.
3) Parálisis pseudobulbar: Contractura y espasmo lingual, disminución de la motilidad de la
lengua. Hay trastorno motor en extremidades con esfínteres normales
4) ESCLEROSIS LATERAL AMINOTROFICA: ES LETAL de 3-5 años comúnmente POR INFECCIÓN
PULMONAR, presentándose con periodos de REMISIÓN Y EXACERBACIÓN DE DIFICULTAD
PARA CAMINAR, TRAGAR , TORPEZA Y FINALMENTE RESPIRAR CON CARÁCTER
AUTOSOMICO DOMINANTE EN GEN 21.
MOTONEURONAS
• Cuales son los signos positivos y negativos que indican lesión a la MOTONEURONA SUPERIOR o síndrome piramidal?
1) Positivos: ESPASTICIDAD, HIPERREFLEXIA en reflejos de estiramiento muscular, clonus, distonía, atetosis y exaltación de
reflejos cutáneos como signo de Babinski
2) Negativos: Debilidad o paresia hemicorporal y pérdida de la destreza, coordinación y control motor.
3) La combinación de ambos afectan la ejecución normal del movimiento con un deterioro en la calidad de vida del paciente.
• Cual es el cuadro clínico del síndrome de la MOTONEURONA INFERIOR?
• R = ATROFIA muscular por denervación, parálisis o PARESIA FLÁCIDA de todos los músculos de la unidad motora afectada,
HIPOTONÍA, hiporreflexia o ARREFLEXIA al interrumpirse la parte eferente del arco reflejo y fasciculaciones producidas por la
actividad espontanea de la unidad motora.
• Cuales son las características clínicas del síndrome de la columna posterior?
1) Se caracteriza por perdida o DISMINUCIÓN DE LA SENSACIÓN DE VIBRACIÓN, SENSACIÓN DE POSICIÓN, DISCRIMINACIÓN DE
DOS PUNTOS, TACTO Y RECONOCIMIENTO DE LA FORMA
2) Estimulos que no eran dolorosos SE CONVIERTEN EN DOLOROSOS y LOS ESTÍMULOS DOLOROSOS SE DESENCADENAN CON
UMBRALES DE ESTIMULACIÓN MAS BAJOS.
• Que caracteriza al síndrome de la arteria espinal anterior?
• R = También conocido como síndrome medular ventral que ocasiona PARAPLEJIA O CUADRIPLEJIA, perdida bilateral de la
sensación de dolor y temperatura por debajo del nivel de la lesión.
• Que estudios realizas para diagnosticar la enfermedad de la motoneurona?
• R = ELECTROMIOGRAFÍA: muestra DENERVACIÓN, EN ESCLEROSIS LATERAL AMINOTROFICA debe presentarse en 3
extremidades mínimo. LINFOMA DE HODKING puede cursar con NEUROPATÍA MOTORA.
• Cual es el tratamiento de lesiones de neuronas motoras?
• R = RICUZOL: disminuye la LIBERACIÓN PRESINAPTICA DE GLUTAMATO haciendo lenta la ELA progresiva.
PLASMAFERESIS logra mejoría. ANTICOLINERGICOS: atropina y amitriptilina
MOTONEURONAS
• - A 53-year-old man complains of clumsiness with both hands, like having difficulty doing up buttons or using his keys. Physical
examination reveals fasciculations of his thigh and forearm muscles; diffuse muscle weakness, loss of muscle bulk, and
increased tone in the upper and lower limbs. There is generalized hyperreflexia, and positive Babinski signs bilaterally. Which of
the following is the most likely natural progression of this condition?
• (A) a long history of remissions and exacerbations (sclerosis lateral aminotrofica)
• (B) sensory loss in the distribution of peripheral nerves
• (C) focal seizures
• (D) a progressively downhill course
• (E) cogwheel rigidity
• - This man has amyotrophic lateral sclerosis (ALS). The disease causes neuronal loss in the anterior horns of the spinal cord and
motor nuclei of the lower brain stem. The disease is one of constant progression, rather than remissions and exacerbations, and
death usually occurs within 5 years. There is no sensory loss and no seizure diathesis, because only the motor system is
involved. There can be signs ofhyperreflexia and spasticity, depending on the balance of upper and lower motor neuron
damage, but not cogwheel rigidity.
• -A 45-year-old man presents with weakness and fasciculations in his arms and legs. His cranial nerves are normal, but there is
weakness of his left handgrip and right leg quadriceps with loss of muscle bulk. There are obvious fasciculations over the left
forearm and right thigh. Tone is increased in the arms and legs and the reflexes are brisk. Which of the following is the most
likely diagnosis?
• (A) amyotrophic lateral sclerosis (ALS)
• (B) myotonic muscular dystrophy
• (C) amyotonia congenita
• (D) tabes dorsalis
• (E) migraine
• - The most common initial symptom of ALS is weakness and wasting of the extremities. The fasciculations can be a very
prominent part of the disease. This is rare in other neurologic disorders.
TRAUMATISMO
• Que te indica el signo de mapache?
• R = Equimosis en parpados debido a fractura de base del craneo por TCE
• Que maniobra resulta útil en el TCE que causa salida de LCR por nariz u oídos (contiene glucosa)?
• R = CONSERVADOR, con ELEVACIÓN DE LA CABEZA, RESTRICCIÓN DE LÍQUIDOS, administración de
acetazolamida EN CASO DE PERSISTENCIA SE REALIZA DRENAJE LUMBAR SUBARACNOIDEO
• Cual es el cuadro clínico de traumatismo en medula espinal?
1) La transección medular completa resulta en PARÁLISIS FLÁCIDA INMEDIATA Y PERDIDA DE LA SENSIBILIDAD
DEBAJO DE LA LESIÓN.
2) Se pierde retención urinaria y fecal.
3) Al final hay paraplejia en las piernas en flexión o extensión.
4) En las LESIONES MENORES los pacientes que dan con DEBILIDAD LEVE DE LAS EXTREMIDADES, TRASTORNOS
SENSITIVOS DISTALES O AMBOS.
• Que caracteriza al síndrome de BROWN-SEQUARD?
• R = Es una LESIÓN UNILATERAL de la MEDULA ESPINAL que da lugar a un TRASTORNO MOTOR IPSOLATERAL con
deterioro concomitante de la PROPIOSEPCION y PERDIDA CONTRALATERAL de las apreciaciones del DOLOR Y
TEMPERATURA
• Cual es el manejo del traumatismo medular?
• R = INMOBILIDAD. GRANDES DOSIS DE ESTEROIDES metilprednisolona 300 mg en bolo seguidos por 5.4
mg/kg/hr x 24 hrs.
TRAUMATISMO
• Cual es la principal etiología de un HEMATOMA SUBDURAL y sus principales características
clínicas?
• R = A subdural hematoma is almost always of VENOUS ORIGIN and secondary to a minor or
severe INJURY TO THE HEAD, but may occur in blood dyscrasias or cachexia in the absence of
trauma. ACUTE subdural hematomas commonly present with a FLUCTUATING LEVEL OF
CONSCIOUSNESS and significant cerebral damage. CHRONIC subdurals may also present with
SEIZURES OR PAPILLEDEMA
• -A 94-year-old man presents with progressive headaches, light-headedness, drowsiness, and
unsteady gait over 6 weeks. On examination, his blood pressure is 160/90 mm Hg, pulse
70/min, lungs clear, and he has no focal weakness. His gait is unsteady but sensation in the
feet is normal. A CT scan reveals a hyperintense clot over the left cerebral cortex. Which of
the following is the most likely cause for this clot?
• (A) is venous in origin
• (B) is arterial in origin
• (C) is from injury to the middle meningeal artery
• (D) is from a subarachnoid hemorrhage
• (E) is from injury to the middle cerebral artery
ESCALA DE GLASGOW MODIFICADA (NIÑOS)
PRUEBA RESPUESTA PUNTUACIÓN
APERTURA DE
OJOS
Espontanea
A órdenes
Al estimulo doloroso
Nula
4
3
2
1
LLANTO COMO
RESPUESTA
VERBAL
 Palabras apropiadas y sonrisas,
fija la mirada y sigue los objetos.
 Tiene llanto, pero consolable.
 Persistente e irritable
 Agitado.
 Sin respuesta.
5
4
3
2
1
RESPUESTA
MOTORA
Obedece ordenes
Localiza el dolor
Retirada ante el dolor
Flexión inapropiada
Extensión
Nula
6
5
4
3
2
1
TCE Leve .... 13-15 puntos
TCE Moderado.. 9-12 puntos
TCE Severo.... 8 puntos o menos
(requiere intubación)
Si EGC: 15-----------------------Consciente
Si EGC: 13-14-------------------Estupor ligero
Si EGC: 11-12-------------------Estupor moderado
Si EGC: 9-10--------------------Estupor profundo
Si EGC: 7-8---------------------Coma superficial
Si EGC: 5-6---------------------Coma moderado
Si EGC: 3-4---------------------Coma profundo
Según el
tamaño
Mioticas  Diámetro < 2mm
Medias  Diámetro 2-5mm
Midriaticas  Diámetro > 5mm
Según relación
entre ellas
Isocoricas  Iguales
Anisocoricas  Desiguales
Discoricas Forma irregular
Reactivas Contracción al foco
luminoso
Según
respuesta
a la luz
Arreactivas
Inmóviles al foco
luminoso
REACTIVIDAD PUPILAR.
Miosis Medias Midriasis
SIRINGOMIELIA
• Que es la siringomielia?
• R = CAVITACIÓN DE LA MEDULA ESPINAL por DESTRUCCIÓN O
DEGENERACIÓN DE LA SUSTANCIA GRIS O BLANCA asociada a la
malformación de ARNOLD CHIARI. Cuenta con 2 etiologías que son
la CONGÉNITA Y ADQUIRIDA. Presenta un síndrome siringomielico
que consiste en:
1) Hipoestesia termoalgesica suspendida disociada
2) PERDIDA DE FUERZA Y AMIOTROFIA
3) ARREFLEXIA
4) Trastornos vegetativos y tróficos
5) Perdida de sensibilidad vibratoria y posicional
SÍNDROME DEL TÚNEL CARPIANO
• A que se asocia el síndrome del túnel carpiano?
• R = HIPOTIROIDISMO, EMBARAZO, diabetes, AR o ACROMEGALIA.
• Cuales son las manifestaciones clínicas del síndrome del túnel carpiano?
1) HORMIGUEO, dolor, ENTUMECIMIENTO EN LA MANO, sobre todo al
comienzo de la mañana y en ocasiones con DEBILIDAD DE LA
MUSCULATURA TENAR con atrofia del abductor corto del pulgar.
2) Puede observarse el SIGNO DEL TINEL que se provoca con un golpesito en
el túnel carpiano REPRODUCIENDO DOLOR Y HORMIGUEO.
• Como dx el sx del túnel carpiano?
• R = CLÍNICA + ELECTROMIOGRAFÍA.
• Cual es el manejo del sx del túnel carpiano?
• R = Descompresión quirúrgica.
COMPRESIÓN DEL NERVIO CUBITAL
• Como se manifiesta clínicamente la compresión del nervio cubital?
1) ADORMECIMIENTO EN EL QUINTO DEDO y en el lado MEDIAL de la mano.
2) El dolor y las parestesias pueden iniciar en el codo e irradiarse a la mano.
3) LAS PARESTESIAS SE EXACERBAN CON LA FLEXIÓN DEL CODO.
• Como diagnosticas la compresión del nervio cubital?
• R = Electrofisiología.
• Cual es el manejo de la compresión del nervio cubital?
• R = Los ESTEROIDES Y LOS AINES. Se recomienda cirugía.
NERVIO CUBITAL
• Cual es la sintomatología en caso de daño al nervio cubital?
• R = Injury to the ulnar nerve results in impaired adduction and abduction of the
fingers. The nerve is commonly injured in ELBOW DISLOCATIONS and fractures. The
fibers arise from the eighth cervical and the first thoracic segments. The ulnar is a
mixed nerve with sensory supply to the medial hand.
• - A 40-year-old man is injured in a car accident and fractures his left elbow. He now
complains of numbness of his fourth and fifth fingers, and weakness in his hand
grip. Neurologic findings confirm weakness of handgrip with weakness of finger
abduction and adduction, and decreased sensation over the fifth finger and lateral
aspect of fourth finger. Which of the following is the most likely diagnosis?
• (A) ulnar nerve injury
• (B) radial nerve injury
• (C) median nerve injury
• (D) carpal tunnel syndrome
• (E) axillary nerve injury
COMPRESIÓN DEL NERVIO RADIAL
• Como se manifiesta la compresión del nervio
radial?
1) “Parálisis de la noche del sábado” por dejar
el brazo colgando después de una
borrachera.
2) MUÑECA FLÁCIDA y debilidad a la extensión
de los dedos.
CIATICO Y TARSO
• Cual es la etiología mas común de la parálisis del
ciático?
• R = INYECCIONES INTRAMUSCULARES profundas
• Que sintomatología da el síndrome del túnel del
tarso?
• R = NERVIO TIBIAL, rama del ciático, cuando se
comprime da origen a DOLOR, PARESTESIAS Y
ENTUMECIMIENTO SOBRE BASE DEL PIE
PREDOMINANDO DURANTE LA NOCHE sin afectar
talón.
MERALGIA PARESTESICA
• Que causa y como se manifiesta clínicamente la meralgia
parestesica?
1) Se debe al atrapamiento del NERVIO CUTÁNEO LATERAL DEL
MUSLO POR DEBAJO DEL LIGAMENTO INGUINAL.
2) Se produce DOLOR URENTE, PARESTESIAS Y
ENTUMECIMIENTO DE LA CARA ANTEROLATERAL DEL
MUSLO.
3) Se presenta en pacientes con EXCESO DE PESO por lo que el
TRATAMIENTO ES BAJAR DE PESO.
4) Nervio FEMOROCUTANEO EXTERNO, se origina en RAÍCES
L2 Y L3 EL CUAL SE PRESIONA EN CASO DE EMBARAZO,
DIABETES Y OBSESOS.
LESIONES DE LOS PARES CRANEALES
• Que datos clínicos se encuentran en el daño al III (OCULOMOTOR) PAR CRANEAL?
1) Frecuentemente causa ANISOCORIA, en la que la pupila mas dilatada denota al LADO AFECTADO.
2) El fenómeno de MARCUS GUNN consiste en que con el ESTIMULO LUMINOSO NO OBTENEMOS
RESPUESTA EN EL LADO AFECTADO después de presentar contracción por efecto de reflejo consensual
al estimular contralateralmente
3) Suele deberse a CAUSAS HIPOXICAS
4) En pacientes con compresión cerebral se mostrara midriasis ipsolateral al sitio afectado como
resultado de compresión del III par craneal en cualquier parte de su recorrido debido a que las fibras
que inervan el esfínter de la pupila discurren en porción periférica del nervio.
5) PTOSIS PALPEBRAL CONTRALATERAL a la lesión.
6) La lesión en este par craneal afecta los MÚSCULOS ELEVADOR DEL PARPADO, recto medial, recto
superior, recto inferior y oblicuo inferior (TODOS MENOS EL RECTO EXTERNO Y EL OBLICUO
SUPERIOR), lo que provoca que los movimientos de ADUCCIÓN, ELEVACIÓN Y DEPRESIÓN DEL OJO
IPSOLATERAL ESTÉN AFECTADOS.
7) Lo anterior provoca que cuando el paciente este de frente el ojo tiende a DESVIARSE HACIA ABAJO Y
AFUERA.
• Que datos clínicos se encuentran en el daño al IV (PATÉTICO O TROCLEAR) PAR CRANEAL?
1) Presentan debilidad del MUSCULO OBLICUO SUPERIOR
2) DIPLOPIA CON ADUCCION Y HACIA ABAJO
• Que datos clínicos se encuentran en el daño al VI (OCULOMOTOR EXTERNO) PAR CRANEAL?
1) Afecta al MUSCULO RECTO LATERAL
2) El ojo afectado se dirige A LA LINEA MEDIA CON INCAPACIDAD DE ABDUCIR EL OJO.
LESIONES DE LOS PARES CRANEALES
• Cual es el cuadro clínico de la parálisis del 3er par craneal ?
• R = Third nerve palsy can result in PTOSIS of the eyelid. There is also loss of the
ability to open the eye, and the EYEBALL IS DEVIATED OUTWARD AND SLIGHTLY
DOWNWARD. With complete lesions, THE PUPIL IS DILATED, DOES NOT REACT TO
LIGHT, and loses the power of accommodation. In diabetes, the pupil is often
spared. The sixth cranial nerve can also be affected by diabetes, but this is much
less common.
• -A 60-year-old man with diabetes acutely develops double vision and discomfort in
his left eye. On examination, there is ptosis of the left eyelid, the eye is rotated
down and out, and the pupil is 3 mm and reactive to light. The right eye is normal.
Which of the following is the most likely diagnosis?
• (A) fourth nerve palsy
• (B) diabetic autonomic neuropathy
• (C) third nerve palsy
• (D) sixth nerve palsy
• (E) seventh nerve palsy
LESIONES DE LOS PARES CRANEALES
• Que datos clínicos se encuentran en el daño al V (TRIGÉMINO) PAR CRANEAL?
1) La parálisis de la primera división (OFTÁLMICA) involucra PERDIDA DE LA SENSIBILIDAD EN LA FRENTE
junto con parálisis del III par y IV.
2) La parálisis de la segunda división (MAXILAR) resulta en la PERDIDA DE SENSIBILIDAD DE LA MEJILLA y se
debe a lesiones del seno cavernoso.
3) La parálisis de la tercera división (MANDIBULAR) resulta en DÉFICIT EN LA MASTICACIÓN
• Que cuadro clínico resulta del síndrome ocasionado por la afección del ganglio de Gasser?
• R = Síndrome caracterizado por DOLOR FACIAL, CEFALEA, PERDIDA DE LA SENSIBILIDAD SENSORIAL, SORDERA Y
PARÁLISIS DEL VI Y VII PARES. EL DOLOR AUMENTA AL MOVER LA MANDÍBULA.
• Cual es el cuadro clínico resultante de la neuralgia del glosofaríngeo?
• R = Es un trastorno unilateral que afecta principalmente a V3 presentándose en forma de ESPASMOS
DOLOROSOS que pueden durar de segundos a minutos y pueden estar asociados a estímulos cutáneos y
auditivos.
• Cual es la etiología de la parálisis de Bell?
• R = Se asocia a HERPES VIRUS
• Que características clínicas tiene la parálisis de Bell?
1) PARÁLISIS DEL VII PAR CON INICIO SÚBITO DE DEBILIDAD FACIAL y pérdida del reflejo acústico.
2) Hay PRÓDROMO VIRAL y adormecimiento o dolor en el oído, lengua o cara.
3) La CUERDA DEL TÍMPANO SE VE ERITEMATOSA y HAY REDUCCIÓN EN LA LAGRIMACIÓN IPSOLATERAL o
salivación.
4) El paciente no puede cerrar los ojos por lo que se debe evitar el daño a la cornea
5) Se diagnostica con ELECTROMIOGRAFÍA denotándose una DISMINUCIÓN EN LA VELOCIDAD DE CONDUCCIÓN
LESIONES DE LOS PARES CRANEALES
• Cual es el manejo de la parálisis de Bell?
• R = VALACICLOVIR o famciclovir y PREDNISONA
• Cual es la característica clínica de la afectación del VIII (VESTIBULOCOCLEAR) par craneal?
1) Porción coclear: El principal síntoma asociado a la lesión o atrofia de esta porción es la HIPOACUSIA
SENSORINEURAL. Se realiza el diagnostico por AUDIOMETRÍA.
2) Porción vestibular: VÉRTIGO POSTURAL PAROXÍSTICO BENIGNO.
• Que características clínicas se observan por daño al XI (ESPINAL ACCESORIO) par craneal?
• R = Dolor significativo acompañado de una DISMINUCIÓN DE LA MOVILIDAD DEL HOMBRO, siendo la
lesión iatrogena la mas común (p.e la BIOPSIA DE LOS GANGLIOS LINFÁTICOS).
• Cuales son los datos clínicos que involucra una lesión en los pares craneales IX Y X (GLOSOFARÍNGEO Y
VAGO)?
1) DISFAGIA, parálisis de la cuerda vocal ipsolateral, así como DEBILIDAD DEL PALADAR Y LA FARINGE.
2) La ELEVACIÓN DEL PALADAR ES ASIMÉTRICA y la ÚVULA SE DESVÍA AL LADO CONTRARIO DE LA
LESIÓN.
3) Se pierde la capacidad sensorial de UN TERCIO POSTERIOR DE LA LENGUA, con disminución en la
sensibilidad en la faringe posterior y laringe.
4) La LESIÓN más común del VAGO es la que involucra al NERVIO LARÍNGEO RECURRENTE, lo que
provoca PARÁLISIS DE LA CUERDA VOCAL IPSOLATERAL Y SUBSECUENTE DISFONÍA.
5) La neuralgia del glosofaríngeo tiene CARACTERÍSTICAS SIMILARES A LA NEURALGIA DEL TRIGÉMINO,
pero es MAS EVIDENTE EN LA LENGUA, LAS AMÍGDALAS, OÍDO Y ANGULO DE LA MANDIBULA
LESIONES DE LOS PARES CRANEALES
• Que características clínicas tiene el daño al XII (HIPOGLOSO) par craneal?
• R = DEBILIDAD en la lengua, FASCICULACIONES y DESVIACIÓN DE LA LENGUA hacia el lado de la lesión.
• Que medicamento es de elección en neuralgia del trigémino y del glosofaríngeo?
• R = CARBAMACEPINA, pero también puede incluir baclofen y fenitoina.
• - A 52-year-old man complains of episodes of severe unilateral, stabbing facial pain that is intermittent for
several hours, and then disappears for several days. The pain is describes as “electric shock-like” and only lasts
a few seconds. Physical examination of the face and mouth is entirely normal. Which of the following
treatments is most effective for this condition?
• (A) morphine
• (B) indomethacin
• (C) cimetidine
• (D) carbamazepine
• (E) lidocaine (Xylocaine) gel
• - This patient has trigeminal neuralgia. Carbamazepine (an anticonvulsant drug) is given in doses varying from
600 to 1200 mg/day. Phenytoin has also been used. The two drugs can also be used in combination. Operative
procedures include alcohol injection of the nerve or ganglion, partial section of the nerve in the middle or
posterior fossa, decompression of the root, and medullary tractotomy.Radiofrequency surgery can destroy pain
fibers but spare motor fibers.
PARALISIS DE BELL
• En que consiste la parálisis de Bell?
• R = Hay PARESIA IDIOPÁTICA DEL FACIAL tipo motoneurona inferior por
inflamación del mismo con probable ETIOLOGÍA VIRAL COMO HERPES
• Cual es el cuadro clínico de la parálisis de Bell?
1) Inicio súbito, empeora con el tiempo, EL DOLOR DEL OÍDO PRECEDE O
ACOMPAÑA A LA DEBILIDAD.
2) CARA RÍGIDA JALADA HACIA UN LADO, RESTRICCIÓN UNILATERAL PARA
CERRAR LOS OJOS con dificultad para comer.
• Cual es el tratamiento de la parálisis de Bell?
• R = 60% SIN TRATAMIENTO, 10% PERMANENTE. PREDNISONA + ACICLOVIR,
se diagnostica con ELECTROMIOGRAFÍA CON DISMINUCIÓN DE LA
CONDUCCIÓN.
VERTIGO
• Cual es la diferencia entre vértigo de origen central y de origen periférico?
• R = In CENTRAL VERTIGO, the vertigo can be MILD AND CHRONIC. In PERIPHERAL disease, THE SYMPTOMS ARE
GENERALLY MORE SEVERE, BUT FINITE (although often recurrent).
• - A 63-year-old woman develops symptoms of nausea, vomiting, and dizziness, which she describes as a to-and-
fro movement of the room like as if she is on a boat. Which of the following findings suggests the vertigo is
central in origin?
• (A) deafness
• (B) symptoms are more protracted but less severe
• (C) unidirectional nystagmus
• (D) visual fixation inhibits vertigo and nystagmus
• (E) spinning sensation is toward the fast phase of nystagmus
• Cuales son las características del vértigo de ORIGEN PERIFÉRICO?
• R = TINNITUS AND DEAFNESS may be found in PERIPHERAL VERTIGO, but NOT CENTRAL. The nystagmus is
usually unidirectional and is never vertical. VISUAL FIXATION INHIBITS VERTIGO AND
NYSTAGMUS during testing in peripheral vertigo
• - A 47-year-old man presents to the emergency room with symptoms of dizziness and difficulty walking. He
describes his dizziness as a spinning sensation of the room with associated nausea and vomiting. Which of the
following findings suggests the vertigo is peripheral in origin?
• (A) optic neuritis
• (B) tinnitus
• (C) bidirectional nystagmus
• (D) vertical nystagmus
• (E) visual fixation does not affect vertigo or nystagmus
VERTIGO
• Cual es la sintomatología del vértigo benigno posicional paroxístico y que maniobra utilizas
para demostrarlo?
• R = Benign paroxysmal positional vertigo (BPPV), which is characterized by sudden-onset
brief episodes of vertigo LASTING LESS THAN A MINUTE. The symptoms are USUALLY
BROUGHT ON BY HEAD MOVEMENT. The cause is commonly attributed to CALCIUM DEBRIS
IN THE SEMICIRCULAR CANALS, KNOWN AS CANALITHIASIS. The debris is loose otoconia
(calcium carbonate) within the utricular sac. Although BPPV can occur after head trauma,
there is usually no obvious precipitating factor. IT GENERALLY ABATES SPONTANEOUSLY AND
CAN BE TREATED WITH VESTIBULAR REHABILITATION. A DIX-HALLPIKE MANEUVER
REPRODUCES HIS SYMPTOMS
• - A 48-year-old man complains of recurrent episodes of sudden-onset dizziness. He notices
an abrupt onset of a spinning sensation when sitting up or lying down in bed. The symptoms
last for 30 seconds and then resolve completely. He has no hearing change or other
neurologic symptoms, and his physical examination is completely normal. A Dix-Hallpike
maneuver reproduces his symptoms. Which of the following is the most likely mechanism
for his vertigo symptoms?
• (A) basilar migraine
• (B) brain stem ischemic events
• (C) benign cerebellar tumors
• (D) calcium debris (calcium carbonate crystals) in the semicircular canals
• (E) Meniere’s disease
ARNOLD CHAIRI
• Que alteraciones se manifiestan en la
malformación de Arnold Chairi?
1) Desplazamiento de las amígdalas cerebelosas
del bulbo raquídeo y del 4to ventrículo hacia
el conducto medular
2) Se puede diagnosticar desde la vida fetal por
medio de us.
EVC
• Cual es la duración de un ataque isquémico transitorio?
• R = Dura < 24 HRS y el DÉFICIT CLÍNICO SE RESUELVE EN MENOS DE 1-2 HRS. Se
debe utilizar ASA COMO PROFILAXIS.
• Cuanto debe medir un infarto lacunar y como se debe ver en la TAC?
• R = Mide <5 MM de diámetro, en la TAC se observan como LESIONES PEQUEÑAS EN
SACABOCADO HIPODENSAS, con RESOLUCIÓN parcial o completa de 4-6 SEMANAS
• Que sintomatología da el EVC en arteria cerebral anterior?
• R = DEBILIDAD y perdida sensitiva en la pierna contralateral y debilidad leve en
brazo.
• Que cambios produce el infarto cerebral anterior?
• R = CAMBIOS CONDUCTUALES y alteración de la MEMORIA.
• Que sintomatología da la obstrucción de la arteria cerebral media?
• R = AFASIA, HEMIPLEGIA CONTRALATERAL, perdida sensitiva, HEMIANOPSIA
HOMÓNIMA, desviación ocular al lado de la lesión.
EVC
• Que sintomatología causa la obstrucción de la ARTERIA CEREBRAL ½ en su PORCIÓN POSTERIOR?
• R = Causa AFASIA DE WERNIKE y HEMIANOPSIA HOMONIMA
• - A 74-year-old woman develops acute neurologic symptoms and presents to the emergency room. An urgent
magnetic resonance imaging (MRI) scan demonstrates acute occlusion in the right posterior cerebral artery. Which of
the following clinical symptoms is she most likely to have?
• (A) homonymous hemianopia
• (B) total blindness
• (C) expressive aphasia
• (D) ataxia and dysarthria
• (E) a right-sided hemiplegia
• -Occlusion of the right posterior cerebral artery is most likely to cause homonymous hemianopia. This artery conveys
blood to the inferior and medial portion of the posterior temporal and occipital lobes and to the optic thalamus.
• - A 63-year-old man developed a transient episode of vertigo, slurred speech, diplopia, and paresthesias. He is
symptom-free now, and clinical examination is entirely normal. His pastmedical history is significant for hypertension
and dyslipidemia. Which of the following is the most likely cause for symptoms?
• (A) posterior circulation transient ischemic attack (TIA)
• (B) anterior communicating artery aneurysm
• (C) hypertensive encephalopathy
• (D) pseudobulbar palsy
• (E) occlusion of the middle cerebral artery
• -Posterior circulation TIA is suggested by the transient episodes. The basilar artery is formed by the two vertebral
arteries and supplies the pons, the midbrain, and the cerebellum. With vertebrobasilar TIAs, tinnitus, vertigo, diplopia,
ataxia, hemiparesis, and bilateral visual impairment are common findings.
EVC
• Cual es el cuadro clínico de la obstrucción de la arteria basilar?
• R = Alteración del estado de alerta, parálisis de los nervios craneales y caídas.
• Como manejas la hipertensión maligna?
• R = Esta entidad se da con una TAD >130, esta se debe REDUCIR A 1/3 pero NUNCA a un VALOR MENOR DE 95
MM HG. NO se debe usar NEFEDIPINO sublingual. En el caso de HIPERTENSIÓN INTRACRANEAL NO SE DEBE
USAR ESTEROIDES.
• Como se procede ante un EVC < 3 hrs?
• R = TROMBOLISIS IV con activador del plasminogeno no tisular. En los que esta CONTRAINDICADA ESTA
TERAPIA son aquellos con:
1) TAS > 185 mm Hg o TAD > 110
2) Sintomatología leve o que mejore de forma rápida
3) CONVULSIONES al inicio de TCE en los 3 MESES PREVIOS
4) CIRUGÍA MAYOR 2 semanas previas
5) Glucemia < 50 o > 400
6) PLAQUETAS < 100,000
7) MUJERES EMBARAZADAS o en lactancia
8) Ingesta actual de anticoagulantes orales con TP > 15
• Cuales son los datos clínicos de una hemorragia intracraneal?
• R = Además de depender de la localización del hematoma suele observarse CEFALEA, CRISIS CONVULSIVAS,
NAUSEA Y VOMITO.
EVC
• Cual es el cuadro clínico de la HEMORRAGIA SUBARACNOIDEA?
1) Se debe principalmente a la ROTURA DE UN ANEURISMASECUNDARIA A UN TRAUMATISMO en pacientes con
ANTECEDENTE DE HAS
2) CEFALEA SUBITA, intensa y pulsátil espontanea o ASOCIADA A ALGÚN ESFUERZO FISICO.
3) El px lo refiere como EL DOLOR MAS INTENSO QUE HA TENIDO EN SU VIDA
4) SE PUEDE ACOMPAÑAR DE NAUSEA, VOMITO, FONOFOBIA Y FOTOFOBIA.
5) Si la evolución es rápida el px puede evolucionar RAPIDAMENTE A ESTADO DE COMA.
6) Este tipo de hemorragia puede presentarse como MUERTE SUBITA.
7) Para su diagnostico se utiliza la TAC, si hay DUDA DX SE REALIZA PUNCIÓN LUMBAR PARA BILIRRUBINA Y CRENOCITOS QUE
CONFIRMAN EL DIAGNOSTICO
8) LA HIPERGLUCEMIA Y EL AUMENTO DE LA TEMPERATURA SE ASOCIAN A MAL PRONÓSTICO.
• Cual es el manejo de la HEMORRAGIA SUBARACNOIDEA?
1) REPOSO ABSOLUTO
2) Monitorización
3) Vendaje de miembros inferiores
4) No estimulación
5) ANALGÉSICOS IV
6) NIMODIPINA ORAL para vasoespasmo cerebral sintomático
7) HIPERVOLEMIA PROFILÁCTICA para prevenir vasoespasmo cerebral
• Cuales son las características de la HEMORRAGIA SUBDURAL?
1) Son hematomas localizados entre la DURAMADRE Y LA ARACNOIDES
2) TCE CON GLASGOW < 8
3) AGUDO < 24 HR, SUBAGUDO >24 HRS Y < 2 SEMANAS Y CRÓNICO > 2 SEMANAS
4) Los síntomas son FOCALIZACIÓN E HIPERTENSIÓN INTRACRANEANA; en las FORMAS SUBAGUDA Y CRÓNICA PREDOMINAN LA
CEFALEA Y DEBILIDAD.
5) Aquellos hematomas que DESPLACEN EL SEPTUM PELUCIDUM > 5 MM de la línea media DEBEN SER EVACUADOS.
EVC
• Cuales son las características de la HEMORRAGIA EPIDURAL?
1) DISMINUCIÓN DEL ESTADO DE ALERTA seguido de
2) INTERVALO DE LUCIDEZ y posteriormente
3) DETERIORO NEUROLÓGICO PROGRESIVO
• Cuales son las características de la trombosis venosa cerebral?
1) 90% presenta como síntoma principal CEFALEA de inicio gradual
2) Debe sospecharse ante un paciente con TAC Y PUNCIÓN LUMBAR NORMAL
3) 40% convulsiones
4) En la TROMBOSIS DEL SENO SAGITAL puede haber DELIRIUM Y AMNESIA.
5) A la exploración física con papiledema y en trombosis venosa focalización
6) El ESTÁNDAR DE ORO PARA EL DIAGNOSTICO ES UNA ANGIOGRAFÍA EN FASE VENOSA ASÍ
COMO RM CON VENOGRAFÍA
7) El tratamiento de elección es la ANTI COAGULACIÓN formal por lo menos SEIS MESES
debido al riesgo de recurrencia así como de tromboembolia pulmonar.
TUMORES SNC
• Cuales son las manifestaciones clínicas de una TUMORACIÓN EN EL LÓBULO FRONTAL?
• R = Declinación progresiva del intelecto y CAMBIOS EN LA PERSONALIDAD.
• Cuales son las manifestaciones clínicas que te da una TUMORACIÓN EN EL LÓBULO
TEMPORAL?
1) Si afectan el TERCIO ANTERIOR se presentara EPILEPSIA Y TRASTORNOS DE LA CONDUCTA
como agresividad e impulsividad
2) SI afecta los 2/3 POSTERIORES se pueden manifestar con HEMIANOPSIA LATERAL
HOMÓNIMA
3) Convulsiones con alucinaciones olfativas o gustativas. Alteraciones en el campo visual.
• Cuales son las manifestaciones clínicas que da una TUMORACIÓN EN EL LÓBULO PARIETAL?
• R = TRASTORNOS CONTRA LATERALES DE LA SENSIBILIDAD. ESTEREOGNOSIA (no
reconocen objetos colocados en la mano), SINDROME DE GERTSMAN
“combinación de alexia, agrafia, acalculia y agnosia digital”.
• Que manifestaciones clínicas da una TUMORACIÓN EN EL LÓBULO OCCIPITAL?
• R = HEMIANOPSIA HOMÓNIMA CRUZADA o defecto del campo visual,
PROSOPAGNOSIA ( no reconoce familiares)
• Que manifestaciones clínicas da la lesión en el TALLO CEREBRAL Y CEREBELOSAS?
• R = PARÁLISIS DE LOS PARES CRANEALES, ATAXIA, incoordinación, nistagmo.
TUMORES DEL SNC
• Cual es el tipo de tumor mas frecuente en la medula espinal?
• R = Ependimoma
• Que es la enfermedad de Von Recklinghausen?
• R = Es una alteración en el CROMOSOMA 17, llamado NEUROFIBROMATOSIS TIPO I que consiste en maculas
MÚLTIPLES HIPERPIGMENTADAS Y NEUROFIBROMAS
• - A 10-year-old boy has multiple tan-colored patches on his skin, and freckle-like skin changes in his armpit
area. The rest of clinical examination is normal. Which of the following conditions is also found in patients
withthis disorder as they get older?
• (A) bilateral eighth nerve tumors
• (B) irregular small pupils
• (C) multiple cutaneous and subcutaneous tumors
• (D) cataracts
• (E) hip involvement
• -The two common forms of neurofibromatosis (NF-1 and NF-2) are genetically distinct. NF-1 is the type with
multiple café au lait spots and is associated with axillary or inguinal freckling, iris hamartomas (Lisch nodules),
peripheral neurofibromas, and bony abnormalities (including kyphoscoliosis). NF-2 is associated with CNS
tumors, particularly bilateral eighth nerve tumors. Skin lesions are spare or absent, and early lens opacities can
occur.
TUMORES DEL SNC• Que caracteriza a la neurofibromatosis tipo II?
• R = Es una alteración en el CROMOSOMA 22. Provoca TUMORES DEL 8vo PAR CRANEAL
• Que es el síndrome de STURGE-WEBER?
• R = ANGIOMA CAPILAR CUTÁNEO congénito que puede ocasionar ANGIOMA DE COROIDES
• Cuales son las características de los astrocitomas (gliomas)?
1) Tumores intracraneales MAS FRECUENTE EN NIÑOS y jóvenes entre 20 y 40ª.
2) EL ASTROCITOMA PILOCITICO es la FORMA MAS BENIGNA de los astrocitomas, PREDOMINA EN NIÑOS y
adultos jóvenes, afectando PRINCIPALMENTE AL CEREBELO.
• Cuales son las características de los OLIGODENDROGLIOMAS?
1) Estos tumores suelen tener zonas de CLACIFICACIONES Y HEMORRAGIAS
2) Tumor de crecimiento lento
• Cuales son las características de los ependimomas?
1) Junto al sistema ventricular REVESTIDO DEL EPÉNDIMO
2) En los niños aparecen principalmente en la primera década de la vida DENTRO DE LOS VENTRÍCULOS.
• Cuales son las características de los germinomas?
• R = Estos tumores tienen muchas características comunes a las neoplasias de células germinales que se
localizan en las gónadas, incluidos los marcadores tumorales
• Cual es el cuadro clínico del meduloblastoma?
1) CRISIS CONVULSIVAS PARCIALES O GENERALIZADAS
2) Síndrome de hipertensión intracraneana caracterizado por la triada de: CEFALEA, VOMITO Y EDEMA DE
PAPILA BILATERAL.
3) Alteraciones cognitivas que se pueden presentar en tumores del lóbulo frontal.
PARKINSON
• Que tratamiento resulta efectivo en el temblor esencial benigno?
• R = PROPANOLOL y si falla primidona o CLONACEPAM
• Que medicamentos causan PARKINSON-LIKE?
• R = Neurolépticos, METOCLOPRAMIDA o reseprina.
• Cual es el neurotransmisor involucrado en Parkinson y cual es la fisiopatologia?
1) DOPAMINA, habiendo un desequilibro con acetilcolina
2) HISTOLOGICAMENTE la enfermedad de Parkinson se caracteriza por DEGENERACIÓN NEURONAL DE LA
SUSTANCIA NIGRA, PARS COMPACTA (PRODUCTORA DE DOPAMINA) y las fibras que la comunica con el
estriado.
3) En el microscopio se encuentran los CUERPOS DE LEWY en la sustancia nigra pars compacta (CUERPOS DE
INCLUSION INTRANEURONAL, EOSINOFILOS CONSTITUIDOS POR ALTAS CONCENTRACIONES DE LA PROTEINA
ALFA-SINUCLEINA).
• Cual es el cuadro clínico del Parkinson?
1) Motoras: TEMBLOR EN REPOSO, RIGIDEZ y bradiscinecia. CON LOS MOVIMIENTOS VOLUNTARIOS SE
ATENUA EL TEMBLOR Y SE EXACERBA CON LAS EMOCIONES. La bradicinecia se manifiesta como lentitud de
movimientos voluntarios. El paciente presenta PERDIDA DE LA EXPRESIÓN FACIAL, menores movimientos de
los labios y lengua al hablar (hipofonia), PERDIDA DE LOS MOVIMIENTOS FINOS DE ESCRITURA (micrografía) y
manipulación de objetos pequeños, DIFICULTAD PARA LEVANTARSE DE UNA SILLA E INICIAR LA MARCHA la
cual se describe como FESTINANTE CON PASOS CORTOS Y RÁPIDOS (APRESURADA).
2) Dermatológica: SEBORREA DE CUERO CABELLUDO y cara con aumento de la sudoración.
3) Psiquiátrica: DEPRESIÓN.
PARKINSON
• Que es la enfermedad de Wilson en el dx diferencial de Parkinson?
• R = Inicio a temprana edad, ANILLOS DE KAYSER-FLEISCHER, hepatitis crónica y aumento de la CONCENTRACIÓN
TISULAR DE COBRE.
• - A 24-year-old man presents with mild jaundice, tremor, and personality changes. Examination reveals
slowness of finger movement, rigidity, and coarse tremor of the outstretched hands. As well there is abnormal
slow movement of the tongue and pharynx resulting in a change in speech and occasional difficulty swallowing.
He is icteric, the liver span is 10 cm, and no spleen is palpable. Which of the following findings is most likely
seen in this condition?
• (A) a reduction of copper excretion in the urine
• (B) an increase of the serum ceruloplasmin content
• (C) no renal involvement
• (D) retention of normal neurologic movements
• (E) a peculiar greenish-brown pigmentation of the cornea
• - In Wilson’s disease, there is usually a reduction of the serum ceruloplasmin content. Signs and symptoms of
injury to the basal ganglia are accompanied by cirrhosis of the liver. Renal involvement is characterized by
persistent aminoaciduria. The most common neurologic finding is tremor. The corneal pigmentation (Kayser-
Fleischer ring) is the most important diagnostic finding on physical examination. If it is absent, any neurologic
findings cannot be ascribed to Wilson’s disease.
• Cual es el estándar de oro en el diagnostico de Parkinson?
1) La NECROPSIA
2) RM donde se observa REDUCCIÓN DE LA SUSTANCIA NEGRA PARS COMPACTA en la porción mesencefalica
superior.
3) PET donde hay MENOR CAPTACIÓN del REACTIVO FLUORO-1-DOPA en el cuerpo estriado.
PARKINSON
• Cual es el tratamiento del Parkinson?
1) LEVODOPA en pacientes MAYORES A 70ª que es AGONISTA DE LA DOPAMINA
utilizada en combinación con CARBIDOPA prevenir la conversión periférica de la
levodopa a dopamina por acción de la ENZIMA DOPADESCARBOXILASA.
2) ANTICOLINERGICOS: en pacientes MENORES A 70ª, CONTRAINDICADOS
EN HPB Y GLAUCOMA DE ÁNGULO CERRADO.
• Que mecanismo de acción tiene la carbidopa?
• R = INHIBE LA ENZIMA QUE CONVIERTE LA LEVODOPA A DOPAMINA
• Cuales son los agonistas de la dopamina utilizados en Parkinson?
• R = BROMOCRIPTINA y pergolida.
• Como actúan los inhibidores de la COMT?
• R = Inhibidores de la Catecolamina-0-Metiltransferasa REDUCEN EL METABOLISMO
DE LA LEVODOPA A 3- METILDOPA y por lo tanto alteran la farmacocinética de la
levodopa en plasma llevando a NIVELES MAS SOSTENIDOS DE DOPAMINA
PLASMÁTICA: TOLCAPONA/ ENTAGAPONA
• Cuales son los fármacos ANTIPSICOTICOS en Parkinson?
• R = RISPERIDONA, clozapina: pudiendo esta causar depleción de MO
PARKINSON
• Que medida quirúrgica se puede utilizar en Parkinson refractario a tx medico?
• R = TALAMOTOMIA o PALIDOTOMIA de 1 solo lado.
• - Which of the following is the most likely finding in a 79-year-old woman with Parkinson’s disease?
• (A) constant fine tremor
• (B) muscle atrophy
• (C) akinesia
• (D) pupillary constriction
• (E) spontaneous remission
• - The characteristic triad in Parkinson’s disease (Tremor, Rigidity, Akinesia) has been expanded to include Postural instability. This
forms the mnemonic TRAP. Autonomic instability is also common. Findings on examination also include masklike facies,
dysarthria, stooped posture, and abnormal gait
• Que fármacos producen Parkinsonismo?
1) FENOTIACINAS, tioxantenos, butiprofenonas, neurolépticos, valproato y fluoxetina.
2) Produciendo SÍNTOMAS PARECIDOS A LOS DEL PARKINSON causando distonias, rigidez, fascies en mascara, bradicinesia y
temblor con menor frecuencia.
3) Los síntomas desaparecen una vez suspendido el fármaco.
• Que entidades conforman al Parkinsonismo plus?
• R = Son síndromes que incluyen SIGNOS Y SÍNTOMAS DE LA ENFERMEDAD DE PARKINSON RELACIONADOS CON
ANORMALIDADES EN PROTEÍNA TAU tales como:
1) Paralisis supranuclear progresiva: Es un trastorno neurodegenerativo que se presenta en la 6ta a 7ma década de la vida y se
caracterizan por rigidez, acinesia o bradicinesia, inestabilidad, caídas, disartria, disfagia y demencia. En la RM se observa
atrofia del coliculo superior en el mesencéfalo. La respuesta al tratamiento con agonistas de dopamina y levodopa es pobre.
2) Degeneracion corticobasal: Los síntomas se desarrollan en la sexta década de la vida incluyendo rigidez focal o asimétrica,
bradicinesia, temblor de acción y reposo y distonía marcada. En la RM se observa perdida cortical localizada en la zona
superior de los lóbulos frontales y parietal contralaterales.
3) Enfermedad difusa de los cuerpos de Lewy: Es un TRASTORNO NEURODEGENERATIVO caracterizado por la presencia de
síntomas parkinsonianos y alteraciones neuropsiquiatricas comúnmente acompañados por DEMENCIA.
HUNTINGTON
• En que gen se ha identificado la enfermedad de Huntington?
• R = Herencia AD CROMOSOMA 4
• Cual es el cuadro clínico de la enfermedad de Huntington?
• R = Aparece entre los 30-50 años. MOVIMIENTOS ANORMALES y CAMBIOS INTELECTUALES CON IRRITABILIDAD,
ESTADO DEL ANIMO, TERMINANDO EN DEMENCIA.
• - A 38-year-old man presents with involuntary facial grimacing, shrugging of the shoulders, and jerking
movements of the limb. His father was similarly affected. There is also a history of mood changes for the past 3
months. On examination, he appears restless with intermittent slow movements of his hands and face. He has
difficulty performing rapid finger movements, and tone is decreased in the upper and lower limbs. Which of
the following is most likely to represent the progression of his illness?
• (A) a normal life span
• (B) a 50% chance of only male children being similarly affected
• (C) mental deterioration
• (D) eventual development of rigidity
• (E) development of hemiparesis
• - This is a case of Huntington’s chorea. It is an autosomal dominant gene (found on the short arm of
chromosome 4), and male and female children are equally affected. Movement disorder, mental deterioration,
and personality change are the hallmarks of the disease, but can be very subtle initially. The disease starts
typically between ages 35 and 40 (although the variation is wide) and runs its course in about 15 years. The
akinetic rigid variety (Westphal variant) of Huntington’s typically has a childhood onset.
HUNTINGTON
• Que datos de TAC se obtienen en corea de Huntington?
• R = Atrofia cerebral y del núcleo caudado
• Cuando hay corea sin antecedentes se duda del dx, pero que causa
la corea de Syndenham?
• R = Es autolimitada por infección por estreptococo del Gpo A
• Que padecimiento AD con datos clínicos similares a CH se debe a
mutación del gen 12 y con ascendencia japonesa?
• R = Atrofia dentorubro palidolisiana con tx igual a CH
• Cual es el neurotransmisor involucrado en corea de hungtinton y
que tratamiento bloqueador de receptores de ese neurotransmisor
controlan la discinecia y cambio conductual?
• R = GABA y se utiliza HALOPERIDOL
HUNTINGTON
• - A 38-year-old man presents with involuntary facial grimacing, shrugging of the shoulders, and jerking movements of the limb. His father was
similarly affected. There is also a history of mood changes for the past 3 months. On examination, he appears restless with intermittent slow
movements of his hands and face. He has difficulty performing rapid finger movements, and tone is decreased in the upper and lower limbs. Which of
the following treatments is helpful in suppressing the movements?
• (A) gamma-aminobutyric acid (GABA)- mimetic agents
• (B) inhibitors of GABA metabolism
• (C) cholinergic agents
• (D) dopamine receptor blockers
• (E) centrally acting cholinesterase inhibitors
• - There are disturbances of norepinephrine, glutamic acid decarboxylase, choline acetyltransferase, GABA, acetylcholine, and somatostatin, but their
significance is poorly understood. Dopamine blocking agents (e.g., haloperidol) can be used to treat psychosis and ameliorate chorea but do not alter
the course of disease. Presynaptic dopamine depletors such as clozapine, reserpine, or tetrabenazine can be used for chorea as well, but have
significant side effects. Antidepressants are helpful for symptomatic treatment. Most patients eventually end up in an institution
• - A 44-year-old man presents with involuntary movements of his face, shoulders, and arms. His father had a similar condition. There is also a history of
mood changes for the past 6 months. On examination, he appears restless with intermittent slow movements of his hands and face. He has difficulty
performing rapid finger movements, and tone is decreased in the upper and lower limbs. Which of the following is the most likely diagnosis?
• (A) Parkinson’s disease
• (B) Huntington’s chorea
• (C) amyotrophic lateral sclerosis (ALS)
• (D) spinal muscular atrophy
• (E) Sydenham’s chorea
• - Huntington’s chorea is the most likely diagnosis given the hereditary nature of this patient’s illness (autosomal dominants). It differs from Sydenham’s
chorea by its gradual onset and slow choreic movements versus brusque jerks seen in Sydenham’s. The caudate nucleus and putamen are both severely
involved in Huntington’s chorea, and degeneration of the caudate nucleus results in enlarged lateral ventricles (with a “butterfly” appearance on CT).
Atrophy is very widespread in the brain and includes the cerebral cortex. Adecrease in glucose metabolism as revealed on positron emission
tomography (PET) scan precedes the evidence of tissue loss.
ALZHEIMER
• Cual es la manifestación principal de la enfermedad de Alzheimer?
• Déficit de ACETILCOLINESTERASA
• La PERDIDA DE MEMORIA a corto plazo
• Cual es la manifestación encontrada en TAC y Biopsia en Alzheimer?
1) Alzheimer’s disease can be quite diffuse, but there is particular INVOLVEMENT OF THE MEDIAL
TEMPORAL LOBES AND CORTICAL ASSOCIATION AREAS.
2) THE ATROPHY OF THE HIPPOCAMPUS IS PARTICULARLY MARKED.
3) Microscopic examination reveals NEUROFIBRILLARY TANGLES AND AMYLOID PLAQUES.
• - A 69-year-old woman presents to the clinic with memory difficulty. The patient’s daughter is
concerned because she is having difficulty doing her finances, such as paying bills. Memory
impairment testing reveals the poor ability to generate lists of words or copy diagrams
(intersecting pentagons). Her remaining physical examination is normal. Which of the following
anatomic findings is most likely with her condition?
• (A) atrophy of the medial temporal lobes
• (B) atrophy of the entire frontal and temporal lobes
• (C) cranial nerve involvement
• (D) transient episodes of hemiplegia
• (E) atrophy of the caudate
DEMENCIA POR CUERPOS DE LEWY
DISTONIA DE TORSIÓN IDEOPATICO
• En que consiste la distonía de torsión idiopático?
• R = En movimientos y posturas diatónicas con antecedentes de nacimiento y
desarrollo normales que inician en la infancia y mas tarde son permanentes
• Cual es el cuadro clínico de la distonía de torsión idiopático?
• R = Movimientos y posturas anormales en paciente con ANTECEDENTES DE
NACIMIENTOS NORMALES. En la infancia comúnmente inicia en piernas.
• Como diagnosticas distonía de torsión idiopático?
1) Exclusión de hipoxia neonatal, Kernicterus, o trauma al nacimiento.
2) Las investigaciones con TAC incluida no revelan alteraciones neurológicas
• Cual es el manejo de la distonía de torsión idiopático?
• R = NO HAY, SE PUEDE ADMINISTRAR LEVODOPA.
DISTONIA DE TORSIÓN FOCAL
• Que caracteriza a la distonía de torsión focal?
• R = Manifestaciones de la distonía de torsión idiopático pero focal que
puede presentarse en la infancia por herencia o sin ella en adulto
• Cuales son las manifestaciones clínicas mas frecuentes de la distonía de
torsión focal?
1) Blefaroespasmo y distonia bucomandibular.
2) Torticolitis espasmódica.
• Que tratamiento resulta útil pero no curativo en la distonia de torsión
focal?
• R = Inyección de toxina botulínica
GILLES DE LA TOURETTE
• Que caracteriza al síndrome de Gilles de la Tourette?
• R = Los TICS MOTORES constituyen el 80% siendo común >21ª
afectando PRINCIPALMENTE LA CARA.
• Cual es el cuadro clínico del síndrome de Gilles de la
Tourette?
1) Tics motores: Olfateo, PARPADEO, fruncir el ceno, ENCOGER
LOS HOMBROS O ASENTIR CON LA CABEZA.
2) Tics fónicos: Murmullos, quejidos, ladridos y aullidos,
coprolalia.
• Cual es el tratamiento de elección del sx de Gilles de la
Tourette?
• R = HALOPERIDOL
ESCLEROSIS MULTIPLE
• Que caracteriza a la esclerosis múltiple?
1) Quizá tenga base autoinmune con desmielinizacion con gliosis reactiva en sustancia blanca encefálica y de la medula espinal
2) Se le ha asociado con Chlamydia pneumoniae, virus herpes 6 y adenovirus
3) El gen HLA-DRB1 en el cromosoma 6p21 es, entre los muchos genes HLA asociados con esclerosis múltiple.
4) El OLIGODENDROCITO encargado de la mielinizacion ES EL BLANCO PRINCIPAL del proceso inflamatorio
principalmente mediado por LT CD 8 CITO TÓXICOS y macrófagos.
5) Patológicamente se distingue por placas de desmielinizacion afectando mas frecuentemente la sustancia blanca
periventricular
• Cual es la clasificación de la esclerosis múltiple?
1) Existen 4 tipos básicos de esclerosis múltiples según su progresión en brote - remisión, primaria progresiva, secundaria
progresiva y progresiva-recurrente.
2) 90% de los pacientes comienza con el TIPO BROTE- REMISIÓN caracterizado por la presentación de uno a dos ataques
por year con remisiones y recuperaciones en los meses posteriores.
• Cual es el cuadro clínico de la esclerosis múltiple?
1) La PRESENTACIÓN COMÚN ES DEBILIDAD, ENTUMECIMIENTO, HORMIGUEO, INESTABILIDAD DE UNA
EXTREMIDAD, paraparesia espástica, NEURITIS RETROBULBAR, trastornos esfinterianos. Los síntomas desaparecen
de días a semanas. En el examen aparecen ATROFIA ÓPTICA, nistagmo, disartria y déficit sensitivo en extremidades.
2) Patrón ataque - recaída - exacerbación, definiéndose ataque como el cuadro agudo de disfunción del SNC que dure mas de
24 hrs en ausencia de fiebre, infecciones o alteraciones metabólicas.
3) FENÓMENO DE UTHOFF, el cual consiste en la aparición o exacerbación de los síntomas tras el ejercicio o un baño con
agua caliente
4) TANTO LAS TAREAS COGNITIVAS COMO LA ACTIVIDAD FÍSICA PROVOCAN CANSANCIO PROFUNDO Y
LOS PACIENTES TARDAN MAS TIEMPO EN RECUPERARSE
5) El CANSANCIO es uno de los síntomas mas comunes y discapacitantes de la enfermedad.
6) La afectación de la vía visual es casi una regla con la presencia de NEURITIS ÓPTICA que se manifiesta como CEGUERA
MONO O BINOCULAR.
7) EL SIGNO DE LHERMITTE (sensación de electricidad que va de la medula hacia las extremidades al flexionar el cuello).
ESCLEROSIS MULTIPLE
• -A 22-year-old woman presents with acute vision loss and pain in the left eye, but no other symptoms. On examination she
appears well, visual acuity is not possible, and she can only perceive movement and bright light. The direct papillary reflex is
absent but the indirect (consensual) response is normal. The optic disc is edematous. Which of the following symptoms is also
most likely present in patients with this condition?
• (A) limb weakness
• (B) hemiplegia
• (C) cervical myelopathy
• (D) sphincter impairment
• (E) seizures
• - This patient has multiple sclerosis (MS). Weakness or numbness in one or more limbs is the initial manifestation of disease in
about half the patients. Other common initial presentations include optic neuritis (25%) and acute myelitis. Hemiplegia,
seizures, and cervical myelopathy (in older patients) occur occasionally as the initial manifestation. Sphincter impairment
usually occurs later in the disease.
• -A 27-year-old woman presents with acute vision loss and pain in the left eye, but no other symptoms. On examination, she
appears well, visual acuity is not possible, and she can only perceive movement and bright light. The direct pupillary reflex is
absent but the indirect (consensual) response is normal. The optic disc is edematous. Which of the following is the most likely
diagnosis?
• (A) diabetic microvascular disease
• (B) arteriosclerosis
• (C) trauma
• (D) multiple sclerosis
• (E) Creutzfeldt-Jakob disease
• -Visual loss in multiple sclerosis varies from slight blurring to no light perception. Other eye symptoms include diplopia and
pain. The classic syndrome of optic or retrobulbar neuritis occurs commonly at some point in the disease, and it is the
presenting symptom in 25% of cases.
ESCLEROSIS MULTIPLE
• Cual es el estudio de elección para dx de esclerosis múltiple?
1) IRM de ENCÉFALO o MEDULA ESPINAL que demuestra MIELOPATIA FOCAL O MÚLTIPLE
EN LA SUSTANCIA BLANCA. Se aplican los criterios de Barkhof y Tintore.
2) El estudio del LCR permite valorar la inflamación intratecal. SE BUSCAN BANDAS
OLIGOCLONALES DE IgG que no están presentes en suero. Un total de PROTEÍNAS
MENOR A 100 Y LEUCOCITOS MENORES A 50 ES LO COMÚN
3) Los POTENCIALES EVOCADOS VISUALES son los mas SENSIBLES Y ESPECÍFICOS, los cuales
DENOTAN DESMIELINIZACION.
• En que consiste la malformación de Arnold Chari, la cual debe descartarse en esclerosis
múltiple?
• R = Observándose la región del agujero magno UNA PARTE DEL CEREBELO Y PARTE INFERIOR
DEL TALLO ENCEFÁLICO SE DESPLAZAN HACIA EL CONDUCTO CERVICAL produciendo déficit
piramidal y cerebeloso en extremidades.
• Cual es el tratamiento de esclerosis múltiple?
• R = INTERFERON B disminuye exacerbaciones al igual que inmunoglobulina IV. NO HAY
FORMA DE DETENER EL AVANCE, se utiliza PREDNISONA EN RECIDIVAS.
• Cual es el pronostico de la esclerosis múltiple?
• R = Mientras mayor sea el numero y volumen de las lesiones peor será el pronostico.
ESCLEROSIS MULTIPLE
• En que consiste el fenómeno de Utoff en
esclerosis múltiple?
• R = EMPEORAMIENTO DE LA VISIÓN al
exponer a una persona a TEMPERATURAS
ELEVADAS O QUE CURSE CON TEMPERATURA
ELEVADA.
EPILEPSIA
EPILEPSIA
ATAXIA DE FRIEDREICH
• Que es la ataxia de Friedreich?
1) AR, en CROMOSOMA 9 q13-9 q21.
2) Es una MARCHA ATÁXICA, manos torpes, respuesta extensora plantar y disminución de ROTS.
3) The pathologic changes are found in the SPINAL CORD TRACTS. Degeneration is seen in the
POSTERIOR COLUMNS, the lateral corticospinal tract, and the spinocerebellar tracts.
4) PIE CAVO BILATERAL
• -A 19-year-old man has had progressive ataxia of gait and great difficulty in running. In the past year, he has
developed hand clumsiness. Physical examination reveals pes cavus, kyphoscoliosis, and both cerebellar and
sensory changes in the legs. There is a positive family history of Friedreich’s ataxia. Where are the pathologic
changes seen in this condition most likely to be found?
• (A) spinal cord tracts
• (B) basal ganglia
• (C) cerebral cortex
• (D) peripheral autonomic nerves
• (E) peripheral motor nerves
• - This young man has Friedreich’s ataxia, associated with a gene defect on chromosome 9. The pathologic
changes are found in the spinal cord tracts. Degeneration is seen in the posterior columns, the lateral
corticospinal tract, and the spinocerebellar tracts. Ataxia, sensory loss, nystagmus, reflex changes, clubfeet, and
kyphoscoliosis are the characteristic findings. The heart is frequently involved, and cardiac disease is a common
cause of death.
DEFICIT VITAMINA E y B12
• Que produce el déficit de vitamina E a nivel SNC?
• R = Se presenta degeneración medulocerebelosa que afecta
principalmente al cordón posterior de la medula espinal.
• Cuales son los datos clínicos de déficit de vitamina E?
• R = Ataxia de extremidades, disminución sensitiva, disminución de ROTS,
confusión y degeneración pigmentaria de la retina.
• Cual es el tratamiento para el déficit de vitamina E?
• R = Alfa tocoferilo
• Que trastorno en el SNC te ocasiona el déficit de vitamina B12?
• Degeneración combinada subaguda de medula espinal.
• Se acompaña de polineuropatía, cambios mentales o neuropatía óptica.
ENCEFALOPATÍA DE WERNICKE
• Que caracteriza la encefalopatía de Wernicke?
• R = Déficit de “TIAMINA B1” y es COMÚN EN BORRACHOS.
• Cual es el cc de la encefalopatía de Wernike?
• R = CONFUSIÓN, ATAXIA Y NISTAGMO que da lugar a oftalmoplejia (parálisis de la mirada
conjugada, debilidad del musculo recto externo)
• -A 43-year-old man is referred from the emergency department with memory loss and difficulty walking. He
was brought in by his wife who has noticed personality changes, truancy from work, and lack of personal care
over the past 2 years. On examination he appears unkempt, smells of urine, and is uncooperative. He cannot
recall the date or season, and gets angry when asked questions. His answers are often fabricated when checked
with his wife. His gait is wide-based, and there is loss of sensation in his feet up to the shins. His motor strength
and reflexes are normal. His ocular movements are normal and there is no nystagmus. In the past he has had
multiple admissions for alcohol withdrawal. Which of the following is the most likely diagnosis?
• (A) Wernicke’s encephalopathy
• (B) Wernicke-Korsakoff syndrome
• (C) Alzheimer’s dementia
• (D) Charcot-Marie-Tooth disease
• (E) vascular dementia
• -The combination of symptoms is typical of chronic alcohol abuse. The mental symptoms are suggestive of
Wernicke-Korsakoff syndrome. Adistal limb sensory-motor neuropathy is also typical of alcoholism. Confusion,
tremulousness, and disorientation are typical for acute alcohol intoxication. Wernicke’s encephalopathy is a
symptom complex of ophthalmoplegia, ataxia, nystagmus, and acute confusional state
DEGENERACIÓN COMBINADA
SUBAGUDA
• Cual es la etiología de la degeneración combinada
subaguda y cual es el cuadro clínico característico?
1) Es causado por la deficiencia de COBALAMINA (VB12)
2) En un inicio encontramos signos y síntomas de lesión de la
COLUMNA POSTERIOR (PARESTESIAS DE MANOS Y PIES,
INESTABILIDAD DE LA BIPEDESTACIÓN Y MARCHA,
ALTERACIONES DE VIBRACIÓN Y POSICIÓN)
3) Después de un tiempo a las 2 semanas posteriores se
identifica una paraparesia atáxica simétrica con
hiperreflexia o hiporreflexia tendinosa y signo de Babinski.
VITAMINAS HIDROSOLUBLES
VITAMINAS LIPOSOLUBLES
ESLEROSIS LATERAL AMINOTROFICA
• Que es la esclerosis lateral aminotrófica?
1) Se refiere a una condición neurología de INICIO EN LA VIDA ADULTA
caracterizada por una DEGENERACIÓN PROGRESIVA DE LAS NEURONAS
MOTORAS SUPERIORES E INFERIORES.
2) La degeneración de las neuronas de la corteza motora se manifiesta como
hiperreflexia, signos positivos de Hoffman, Babinski y clonus.
3) El termino esclerosis lateral se refiere a la DUREZA CUANDO SE PALPAN LAS
COLUMNAS LATERALES DE LA MEDULA ESPINAL en las autopsias de pacientes
con esta enfermedad.
4) El termino aminotrófica se refiere a la ATROFIA MUSCULAR, DEBILIDAD Y
FASCICULACION debidas a la degeneración de las neuronas motoras inferiores.
• Que hallazgos muestra la electromiografía al respecto de la esclerosis lateral
aminotrófica?
• R = Demuestra DENERVACIÓN de POR LO MENOS 3 EXTREMIDADES.
• Cual es el manejo de la esclerosis lateral aminotrófica?
• R = RILUZOLE QUE ES UN ANTAGONISTA DEL GLUTAMATO Y LA VITAMINA E.
Deben ser tratados como si tuvieran lesión de la columna cervical hasta que se
demuestre lo contrario.
CHARCOT-MARIE-TOOTH
• Cuales son las características de la mononeuropatia
hereditaria de Charcot-Marie-Tooth?
1) Carácter AD en CROMOSOMA 17.
2) Clínicamente con DEFORMIDADES DEL PIE,
TRASTORNOS DE LA MARCHA.
3) El electrodiagnóstico muestra DISMINUCIÓN DE LA
CONDUCCIÓN MOTORA Y SENSITIVA (neuropatía
motora y sensitiva hereditaria tipo I) NMSH tipo I.
4) NMSH tipo II disminuye solo un poco.
5) Histológicamente hay PERDIDA AXONAL.
DEJEWINE-SOTTAS NMSH TIPO III
• Que caracteriza a la mononeuropatía
hereditaria Dejewine-Sottas NMSH tipo III?
1) AR, polineuropatía progresiva motora y
sensitiva con debilidad, ataxia, disminución
sensitiva e hiporreflexia.
2) Histopatológicamente con NERVIOS
PERIFÉRICOS SENSIBLES Y PALPABLES,
desmielinizacion segmentaria, HIPERPLASIA
DE CÉLULAS DE SCHWAN.
ENFERMEDAD DE REFSUM NMSH TIPO IV
• Que caracteriza a la mononeuropatia hereditaria
de la enfermedad de Refsum NMSH tipo IV?
1) AR, trastorno en el metabolismo del ACIDO
FITANICO.
2) Hay DEGENERACIÓN RETINIANA PIGMENTARIA
+ polineuropatia sensitivomotora.
3) DISFUNCIÓN AUDITIVA, MIOCARDIOPATÍA.
4) El tratamiento es LA DISMINUCIÓN DEL ACIDO
TIFANICO.
PORFIRIA
• En la porfiria aguda con manifestaciones en
los parámetros motores y debilidad proximal,
que medicamento resulta útil?
1) HEMATINA.
2) Administración alta de CHBTS, glucosa IV.
3) Propanolol que controla la taquicardia e
hipertensión.
LYME
• Cuales son las manifestaciones neurológicas
de la enfermedad de Lyme?
1) Meningitis,
2) Poliradiculopatia,
3) Mononeuropatia multiple y
4) Neuropatía cutánea.
SARCOIDOSIS
• Cuales son las manifestaciones neurológicas
de la sarcoidosis?
• R = Parálisis de los pares craneales, en especial
el 7 que responde a esteroides
SÍNDROME DE GUILLAIN-BARRE
• Cual es la etiología del síndrome de Guillain-Barre?
1) Suele asociarse a una infección de vías respiratorias o gastrointestinal.
2) Principalmente el CAMPYLOBACTER JEJUNI Y CMV.
• Cuales son los signos y síntomas de la poliradiculoneuropatia progresiva
aguda/sub aguda conocido como síndrome de Guillain- Barre?
• R = Debilidad ascendente de comienzo en piernas.
• Cuales son los estudios para el diagnostico de síndrome de Guillain- Barre?
1) LCR con AUMENTO DE PROTEÍNAS
2) Células normales
3) Electrofisiología con alteraciones
4) Histología que muestra DESMIELINIZACION PRIMARIA
• Cual es el manejo del síndrome de Guillain-Barre?
• R = Administración de INMUNOGLOBULINA IV O PLASMAFERESIS.
MIASTENIA GRAVE
• Cual es el cuadro clínico de la miastenia grave?
1) DIPLOPÍA, debilidad de músculos oculares, debilidad progresiva pudiendo
causar parálisis respiratoria, PTOSIS.
2) El cuadro clínico confirma DEBILIDAD Y FATIGA DE MÚSCULOS EL CUAL
MEJORA CON EL REPOSO.
• Como se realiza el diagnostico de miastenia grave?
1) Respuesta a ANTICOLINESTERASA de acción corta con EDROFONIO con
vida media de 1 minuto en el cual los PACIENTES CON RESPUESTA TIENEN
MEJORÍA NOTORIA DURANTE 5 MINUTOS.
2) NEOSTIGMINA, la respuesta dura 2 hrs, con sulfato de atropina en caso de
efectos muscariniocos adversos.
3) ELECTROFISIOLOGIA que muestra DISMINUCIÓN DE LA
TRANSMISIÓN.
4) LAB con Ac VS RECEPTOR DE ACETILCOLINA
• Cual es el tratamiento de miastenia grave?
• R = NEOSTIGMINA. Timectomia en caso de timoma. PROSCRITOS
AMINOGLUCOSIDOS por que causan bloqueo neuromuscular
MIASTENIA GRAVE
• - A 30-year-old woman complains of double vision, and easy fatigue with
exercise. The fatigue improves with resting, but it is interfering with her work.
Examination reveals ptosis and impaired eye movements with normal pupillary
response. The double vision is brought out by asking her to look at the ceiling, and
after a sustained interval, the eyes slowly drift down. Which of the following is the
most likely diagnosis?
• (A) optic atrophy
• (B) ophthalmic zoster
• (C) paralysis agitans
• (D) Horner syndrome
• (E) myasthenia gravis
• - In myasthenia gravis, weakness of the facial and levator palpebrae muscles
produces a characteristic expressionless face, with drooping of the eyelids.
Weakness of the ocular muscles may cause paralysis or weakness of individual
muscles, paralysis of conjugate gaze, ophthalmoplegia, or a pattern similar to
internuclear ophthalmoplegia. The presence of normal pupillary responses to light
and accommodation with weakness of extraocular muscles, levators, and
orbicularis oculi is almost completely diagnostic of myasthenia
SÍNDROME DE LAMBERT-EATON O
MIASTENIFORME
• Que puede ocasionar el síndrome de lambert-eaton o miasteniforme?
1) En este hay liberación DEFECTUOSA DE ACETILCOLINA.
2) Se presenta con DEBILIDAD DE MÚSCULOS PROXIMALES y se
diferencia de miastenia gravis por que LA FUERZA SE AUMENTA
CON CONTRACCIÓN SOSTENIDO.
• Como diagnosticas el síndrome de lambert-eaton?
• R = Electrofisiología por AUMENTO DE LA RESPUESTA
MUSCULAR A LA ESTIMULACIÓN
• Cual es el manejo del síndrome de lambert-eaton?
• R = PLASMAFERESIS, inmunodepresores (azatropina, prednisona).
NEUROINFECCION• Ayuda la toma de LCR en el caso de absceso cerebral?
• R = NO, de echo ESTA PROSCRITO por que puede causar herniación.
• Cual es la complicación mas frecuente del SIDA en el SNC?
• R = This is one of the most common neurologic complications of AIDS. Its pathologic substrate is
degeneration of the spinal tracts in the posterior and lateral columns, which have a vacuolated
microscopic appearance. Although the morphologic changes and clinical manifestations are similar
to those associated with vitamin B12 deficiency, the pathogenetic mechanism is probably not
related to dietary deficiencies. Since there is no specific clinical or laboratory test available for the
diagnosis of this syndrome, VACUOLAR MYELOPATHY IN AIDS PATIENTS REMAINS A DIAGNOSIS OF
EXCLUSION. This implies that other HIV-related neurologic complications must be ruled out
• -A 56-year-old man is brought to the emergency department by his wife because of memory loss
and difficulty walking. She has noticed personality changes, truancy from work, and lack of
personal care over the past 1 year. On examination he appears unkempt, smells of urine, and is
uncooperative. He cannot recall the date or season,and gets angry when asked questions. His
answers are often fabricatedwhen checked with his wife. The blood pressure is 150/90 mm Hg,
pulse 100/min, and he is diaphoretic and tremulous. His gait is wide based, and motor strength and
reflexes are normal. His ocular movements are normal but there is nystagmus on lateral gaze. In the
past he has had multiple admissions for alcohol withdrawal. Which of the following is the most
appropriate next step in management?
• (A) prophylactic phenytoin administration
• (B) prophylactic diazepam administration
• (C) prophylactic carbamazepine administration
• (D) calcium administration
• (E) steroid administration
• - Prophylactic administration of diazepam in a withdrawing alcoholic can prevent or reduce severe
syndromes such as delirium tremens (DTs). Prophylactic phenytoin, however, is not helpful. Acalm,
quiet environment with close observation and frequent reassurance is very important. Vitamin
administration (especially thiamine) is important, but frequently, severe magnesium depletion
slows improvement.
LCR
BOTULISMO
• Cual es la fisiopatología del botulismo?
• R = La toxina EVITA LA LIBERACIÓN DE ACETILCOLINA en las uniones
neuromusculares y sinapsis autónomas, transmitida por COMIDAS ENVASADAS
• Cual es el cuadro clínico del botulismo?
1) Desarrollo SÚBITO DE DEBILIDAD INTENSA DESPUÉS DE 72 HRS DE LA COMIDA.
2) El típico cuadro es de DIPLOPÍA, VISIÓN BORROSA, midriasis no reactiva,
estreñimiento por íleo paralitico, PTOSIS, debilidad facial, DIFICULTAD
RESPIRATORIA Y DEBILIDAD EN EXTREMIDADES INFERIORES CON SENSIBILIDAD
CONSERVADA.
• Como se dx el botulismo?
• R = SUERO CON TOXINA. Electrofisiológico con estimulación repetida con aumento
de la respuesta muscular de manera progresiva
• Cual es el manejo del botulismo?
1) ANTITOXINA en pacientes NO ALÉRGICOS AL SUERO DE CABALLO.
2) GUANIDINA facilita la liberación de acetilcolina
MIGRANA
• Cual es el cuadro clínico de la migraña?
1) Cefalea intensa, pulsátil, unilateral en la región frontal o temporal.
2) La migraña con aura consiste en un inicio con alteraciones visuales con
luces-centilleos o luces de colores, palidez, vértigo, alteraciones
sensitivas o debilidad unilateral,
3) Afasia transitoria o alteraciones en el lenguaje que aparecen
previamente al establecimiento de la migraña.
• A que se refiere el termino estatus migranoso?
• R = Se refiere a pacientes ya conocidos con migraña con manifestaciones
clínicas por mas de 72 Hrs
• Cual es el manejo de la migraña?
• R = Triptanos y derivados de la ergotamina
MIGRANA
• Cuales son las 2 variantes de migraña a considerar?
1) Los síntomas prodrómicos son el resultado de una disfunción en el territorio de la circulación
cerebral posterior con sintomatología visual bilateral, ataxia, disartria, vértigo, parestesias de las
extremidades e incluso debilidad. Puede haber perdida de la conciencia antes de comenzar la
cefalea.
2) Migraña hemipléjica: Es poco frecuente e incluye hemiplejia, que puede persistir días después de
calmarse la cefalea.
• - An 18-year-old woman has periodic episodes that begin with severely decreased vision, followed
by ataxia, dysarthria, and tinnitus. The symptoms last for 30 minutes and are then followed by a
throbbing occipital headache. Which of the following is the most likely diagnosis?
• (A) vertebral-basilar insufficiency
• (B) chronic basilar artery dissection
• (C) classic migraine
• (D) ophthalmoplegic migraine
• (E) basilar migraine
• Basilar migraine can be very dramatic, and can resemble ischemia in the territory of the basilar
posterior cerebral arteries. The visual symptoms of basilar migraine typically affect the whole of
both visual fields, and can even cause temporary cortical blindness. There can also be an alarming
period of coma or quadriplegia.
CEFALEA TENSIONAL
• Cual es el cuadro clínico de la cefalea tensional?
1) Dolor difuso, en banda, de carácter sordo, que
puede acompañarse de hiperestesia en el cuero
cabelludo y agravarse con el ruido y la luz.
2) Típicamente la distribución del dolor es en
forma de banda, bilateral y se extiende desde la
frente hacia los temporales y nuca.
3) Como profilaxis pueden utilizarse antidepresivos
triciclicos y la amitriptilina es la más eficaz.
CEFALEA EN RACIMOS O ENFERMEDAD DE
HORTON
• Que caracteriza a la cefalea en racimos o
enfermedad de Horton?
1) Intenso dolor unilateral periorbitario
acompañado de inyección conjuntival, lagrimeo,
rinorrea y síndrome de Horner
2) El comienzo del dolor suele ser en la madrugada
o al despertar muy temprano.
3) Para su tratamiento son útiles los fármacos
utilizados en migraña.
4) Nifedipino y berapamil como parte del manejo
profiláctico.
NARCOPLEPSIA• Cual es la sintomatología en caso de narcolepsia?
• R = The early age of onset and otherwise good health suggest a diagnosis of narcolepsy, which is usually accompanied by other
symptomatology. Hypnagogic hallucinations are almost always visual. They occur most frequently at the onset of sleep, either
during the day or at night. They are generally very vivid. Cataplexy is a brief loss of muscle power without loss of consciousness.
The patient is fully aware of what is going on. The paralysis may be complete or partial.
• -A 31-year-old woman complains of excessive sleepiness during the daytime for years despite adequate nighttime sleep. She
has episodes of intense drowsiness three to four times a day, even when at work or while eating meals. She has sought medical
attention in the past, after falling asleep while driving. She is slender and otherwise healthy and on no medications. Which of
the following treatments is most likely indicated for her condition?
• (A) a device providing continuous positive airway pressure (CPAP) at night
• (B) oral surgery
• (C) tracheostomy
• (D) amphetamines
• (E) benzodiazepines at bedtime
• -This woman does not have risk factors for sleep apnea (older age, snoring, obesity) and likely has narcolepsy. Adrenergic
stimulant drugs such as methylphenidate or amphetamines help the sleepiness, and tricyclic compounds can help the cataplexy.
Strategically planned naps can also be helpful.
• -A 25-year-old man complains of excessive sleepiness during the daytime for years despite adequate nighttime sleep. He has
sought medical attention after falling asleep while driving. He is slender and otherwise healthy and on no medications. Which of
the following symptoms might he also complain about?
• (A) excessive snoring (wife’s report)
• (B) automatic behavior (wife’s report)
• (C) restless sleep (wife’s report)
• (D) paresthesias
• (E) morning headache
• -Automatic behavior with amnesia is a common manifestation of the narcolepsycataplexy syndromes, occurring in 50% of cases.
Automatic behavior can be confused with complex partial seizures. Paresthesias are not part of narcolepsy syndrome. Snoring,
restless sleep, and morning headache suggest sleep apnea.
DISTROFIAS MUSCULARES
DISTROFIA MIOTONICA
• Cual es la etiología de la distrofia miotonica?
• R = Trastorno AD lentamente progresivo entre los 30-40, en ocasiones durante la
infancia, localizado en el cromosoma 19.
• Cual es el cuadro clínico de la distrofia miotonica?
1) Molestias de rigidez muscular con DISMINUCIÓN DE LA RELAJACIÓN DESPUÉS DE
LA CONTRACCIÓN.
2) Hay debilidad y emaciación de músculos faciales, esternocleidomastoideos y
distales.
3) Cataratas, calvicie, atrofia testicular y DM
• Como dx distrofia miotonica?
• R = Electromiografía revela descargas miotonicas adicionales.
• Cual es el manejo de la distrofia miotonica?
• R = Sulfato de quinidina, procainamida o fenitoina.
MIELOPATIA POR VIRUS DE LA LEUCEMIA
DE LINFOCITOS T HUMANOS
• Como se transmite la mielopatia por virus de la
leucemia de linfocitos T humanos?
• R = El virus de la leucemia de linfocitos T humanos
HTLV-1 se transmite por medio del seno materno,
sexual, sangre y agujas contaminadas.
• Cuales son los datos de laboratorio que orientan a
mielopatia por virus de la leucemia de linfocitos T
humanos?
• R = Suero y LCR hay AC vs HTLV-1, datos similares a
esclerosis multiple en LCR, IRM o EEG. NO HAY TX
ESPECIFICO.
MIOTONIA CONGENITA
• Cual es la etiología de la miotonia congénita?
1) Se hereda de forma AD en cromosoma 19.
2) Se caracteriza por miotonia generalizada sin
debilidad en movimiento, hay rigidez muscular
que aumenta con el frio y disminuye con el
ejercicio.
3) Hipertrofia muscular.
• Cual es el manejo de la miotonia congénita?
• R = Sulfato de quinidina, procainamida
MIOSITIS DE CUERPOS DE INCLUSIÓN
• Cual es la etiología de la miositis de cuerpos de
inclusión?
1) Origen desconocido.
2) Cuadro clínico comienza arriba de los 20ª con
debilidad proximal progresiva, 1ero de extremidades
inferiores y después de extremidades superiores.
• Cual es el tratamiento de la miositis de cuerpos de
inclusión?
• R = Inmunoglobulina IV, los esteroides son ineficaces
MIOPATÍAS MITOCONDRIALES
• Cuales son las características de las miopatías
mitocondriales?
• R = En la biopsia con tinción modificada de Gomon, se
observan fibras rojas que contienen un cumulo de
mitocondias anormales.
• Cual es el cuadro clínico de las miopatías mitocondriales?
1) Oftalmoplejia progresiva, debilidad de las extremidades
exacerbada por actividad.
2) Otros con epilepsia mioclonica o combinación de miopatía
encefalopatía, acidosis láctica.
3) Episodios similares al EVC
SÍNDROME DE PARALISIS PERIODICA
• Cual es la etiología del síndrome de parálisis periódica?
• R = Herencia AD, gen 17 en canal de Na.
• Cual es el cuadro clínico del síndrome de paralisis periodica?
1) Episodios de debilidad o parálisis flácida acompañándose
ocasionalmente de hipokalemia.
2) Suele presentarse al despertar, después del ejercicio.
• Cual es el manejo del síndrome de parálisis periódica?
• R = Dieta baja en CHBTS, acetazolamida, Cloruro de K VO o IV.
Glucosa e insulina.
Consejos
1. Leer diariamente y en bloques
2. Adiós Partys un tiempo
3. Has ejercicio y come bien durante el estudio
4. Toma algún curso bueno si tienes la posibilidad
5. Ten Fe.
BIBLIOGRAFIA
• EXARMED
• PAPADAKIS
• CTO
• HARRISON
• AMIR
• USMLE STEPS

Más contenido relacionado

La actualidad más candente (20)

neuropatia diabetica
neuropatia diabeticaneuropatia diabetica
neuropatia diabetica
 
Semiologia motora
Semiologia motoraSemiologia motora
Semiologia motora
 
Pericarditis aguda
Pericarditis agudaPericarditis aguda
Pericarditis aguda
 
Expo pie diabetico guia alad 2014 USC
Expo pie diabetico guia alad 2014 USCExpo pie diabetico guia alad 2014 USC
Expo pie diabetico guia alad 2014 USC
 
ESTENOSIS MITRAL
ESTENOSIS MITRALESTENOSIS MITRAL
ESTENOSIS MITRAL
 
Neuronamotora Superior e Inferior
Neuronamotora Superior e InferiorNeuronamotora Superior e Inferior
Neuronamotora Superior e Inferior
 
Estenosis AóRtica
Estenosis AóRticaEstenosis AóRtica
Estenosis AóRtica
 
Endocarditis
EndocarditisEndocarditis
Endocarditis
 
Sindrome de neurona motora
Sindrome de neurona motora Sindrome de neurona motora
Sindrome de neurona motora
 
Esclerosis lateral amiotrófica
Esclerosis lateral amiotróficaEsclerosis lateral amiotrófica
Esclerosis lateral amiotrófica
 
ENDOCARDITIS
ENDOCARDITISENDOCARDITIS
ENDOCARDITIS
 
Síndrome demencial clase
Síndrome demencial claseSíndrome demencial clase
Síndrome demencial clase
 
Neuropatía diabetica
Neuropatía diabetica  Neuropatía diabetica
Neuropatía diabetica
 
Polineuropatias
Polineuropatias Polineuropatias
Polineuropatias
 
Guías Europeas Marcapasos y Resincronizadores
Guías Europeas Marcapasos y ResincronizadoresGuías Europeas Marcapasos y Resincronizadores
Guías Europeas Marcapasos y Resincronizadores
 
Electrolitos
Electrolitos Electrolitos
Electrolitos
 
8. motoneurona superior e inferior
8. motoneurona superior e inferior8. motoneurona superior e inferior
8. motoneurona superior e inferior
 
Status epileptico
Status epilepticoStatus epileptico
Status epileptico
 
Esclerosis multiple
Esclerosis multiple Esclerosis multiple
Esclerosis multiple
 
NERVIOS CRANEALES
NERVIOS CRANEALESNERVIOS CRANEALES
NERVIOS CRANEALES
 

Similar a 8.NEUROLOGIA

2. Neuroanatomía y semiología en Neurocirugía.pdf
2. Neuroanatomía y semiología en Neurocirugía.pdf2. Neuroanatomía y semiología en Neurocirugía.pdf
2. Neuroanatomía y semiología en Neurocirugía.pdfLuisUrdialesCastillo
 
esclerosislateralamiotrficapatologiaok-141013084006-conversion-gate01.pptx
esclerosislateralamiotrficapatologiaok-141013084006-conversion-gate01.pptxesclerosislateralamiotrficapatologiaok-141013084006-conversion-gate01.pptx
esclerosislateralamiotrficapatologiaok-141013084006-conversion-gate01.pptxEFGS
 
Manifestaciones clinicas sindrome del tronco cerebral
Manifestaciones clinicas sindrome del tronco cerebralManifestaciones clinicas sindrome del tronco cerebral
Manifestaciones clinicas sindrome del tronco cerebralGiorjhian Lomas Arenas
 
esclerosislateralamiotrficapatologiaok-141013084006-conversion-gate01.pdf
esclerosislateralamiotrficapatologiaok-141013084006-conversion-gate01.pdfesclerosislateralamiotrficapatologiaok-141013084006-conversion-gate01.pdf
esclerosislateralamiotrficapatologiaok-141013084006-conversion-gate01.pdfteresachimbo
 
Evento cerebro vascular - Medicina Interna
Evento cerebro vascular - Medicina InternaEvento cerebro vascular - Medicina Interna
Evento cerebro vascular - Medicina InternaDavid Cortez
 
Enfermedades de Neurona Motora
Enfermedades de Neurona MotoraEnfermedades de Neurona Motora
Enfermedades de Neurona MotoraNorma Obaid
 
Principales sindromes del sitema nervioso
Principales sindromes del sitema nerviosoPrincipales sindromes del sitema nervioso
Principales sindromes del sitema nerviosoBergoglio
 
Exploración neurológica
Exploración neurológicaExploración neurológica
Exploración neurológicaIk Hanhemaniano
 
Técnica de exploración sensibilidad
Técnica de exploración sensibilidadTécnica de exploración sensibilidad
Técnica de exploración sensibilidadHugo Otaku
 
RESUMEN DE SÍNDROMES LOBARES, CARRERA DE MEDICINA
RESUMEN DE SÍNDROMES LOBARES, CARRERA DE MEDICINARESUMEN DE SÍNDROMES LOBARES, CARRERA DE MEDICINA
RESUMEN DE SÍNDROMES LOBARES, CARRERA DE MEDICINAMaria Paula Pecelis
 
22. evc isquemico j dr. fabián yungán
22. evc isquemico j dr. fabián yungán22. evc isquemico j dr. fabián yungán
22. evc isquemico j dr. fabián yungánDr. Fabián Yungán
 
Lesiones del cerebelo ok
Lesiones del cerebelo okLesiones del cerebelo ok
Lesiones del cerebelo okGustavo Lozano
 
2da 3era clase examen neurologico espinal.ppt
2da 3era clase examen neurologico espinal.ppt2da 3era clase examen neurologico espinal.ppt
2da 3era clase examen neurologico espinal.pptJUANPABLO510869
 
Manejo del paciente en coma
Manejo del paciente en comaManejo del paciente en coma
Manejo del paciente en comaTARIK022
 
preguntashhhhhhhhhhhhhhhhhhhhhhhhhhhhhhhhhhhhhhhhhhhh
preguntashhhhhhhhhhhhhhhhhhhhhhhhhhhhhhhhhhhhhhhhhhhhpreguntashhhhhhhhhhhhhhhhhhhhhhhhhhhhhhhhhhhhhhhhhhhh
preguntashhhhhhhhhhhhhhhhhhhhhhhhhhhhhhhhhhhhhhhhhhhhsebastian1407
 

Similar a 8.NEUROLOGIA (20)

Trauma Medular
Trauma MedularTrauma Medular
Trauma Medular
 
2. Neuroanatomía y semiología en Neurocirugía.pdf
2. Neuroanatomía y semiología en Neurocirugía.pdf2. Neuroanatomía y semiología en Neurocirugía.pdf
2. Neuroanatomía y semiología en Neurocirugía.pdf
 
esclerosislateralamiotrficapatologiaok-141013084006-conversion-gate01.pptx
esclerosislateralamiotrficapatologiaok-141013084006-conversion-gate01.pptxesclerosislateralamiotrficapatologiaok-141013084006-conversion-gate01.pptx
esclerosislateralamiotrficapatologiaok-141013084006-conversion-gate01.pptx
 
Manifestaciones clinicas sindrome del tronco cerebral
Manifestaciones clinicas sindrome del tronco cerebralManifestaciones clinicas sindrome del tronco cerebral
Manifestaciones clinicas sindrome del tronco cerebral
 
esclerosislateralamiotrficapatologiaok-141013084006-conversion-gate01.pdf
esclerosislateralamiotrficapatologiaok-141013084006-conversion-gate01.pdfesclerosislateralamiotrficapatologiaok-141013084006-conversion-gate01.pdf
esclerosislateralamiotrficapatologiaok-141013084006-conversion-gate01.pdf
 
Evento cerebro vascular - Medicina Interna
Evento cerebro vascular - Medicina InternaEvento cerebro vascular - Medicina Interna
Evento cerebro vascular - Medicina Interna
 
Enfermedades de Neurona Motora
Enfermedades de Neurona MotoraEnfermedades de Neurona Motora
Enfermedades de Neurona Motora
 
Principales sindromes del sitema nervioso
Principales sindromes del sitema nerviosoPrincipales sindromes del sitema nervioso
Principales sindromes del sitema nervioso
 
Exploración neurológica
Exploración neurológicaExploración neurológica
Exploración neurológica
 
Abordaje coma
Abordaje comaAbordaje coma
Abordaje coma
 
Técnica de exploración sensibilidad
Técnica de exploración sensibilidadTécnica de exploración sensibilidad
Técnica de exploración sensibilidad
 
Coma semio neuro
Coma  semio neuroComa  semio neuro
Coma semio neuro
 
Choque medular
Choque medularChoque medular
Choque medular
 
RESUMEN DE SÍNDROMES LOBARES, CARRERA DE MEDICINA
RESUMEN DE SÍNDROMES LOBARES, CARRERA DE MEDICINARESUMEN DE SÍNDROMES LOBARES, CARRERA DE MEDICINA
RESUMEN DE SÍNDROMES LOBARES, CARRERA DE MEDICINA
 
22. evc isquemico j dr. fabián yungán
22. evc isquemico j dr. fabián yungán22. evc isquemico j dr. fabián yungán
22. evc isquemico j dr. fabián yungán
 
Lesiones del cerebelo ok
Lesiones del cerebelo okLesiones del cerebelo ok
Lesiones del cerebelo ok
 
Esclerosis lateral amiotrófica.
Esclerosis lateral amiotrófica.Esclerosis lateral amiotrófica.
Esclerosis lateral amiotrófica.
 
2da 3era clase examen neurologico espinal.ppt
2da 3era clase examen neurologico espinal.ppt2da 3era clase examen neurologico espinal.ppt
2da 3era clase examen neurologico espinal.ppt
 
Manejo del paciente en coma
Manejo del paciente en comaManejo del paciente en coma
Manejo del paciente en coma
 
preguntashhhhhhhhhhhhhhhhhhhhhhhhhhhhhhhhhhhhhhhhhhhh
preguntashhhhhhhhhhhhhhhhhhhhhhhhhhhhhhhhhhhhhhhhhhhhpreguntashhhhhhhhhhhhhhhhhhhhhhhhhhhhhhhhhhhhhhhhhhhh
preguntashhhhhhhhhhhhhhhhhhhhhhhhhhhhhhhhhhhhhhhhhhhh
 

Más de Alejandro Sandoval (13)

Hipertension en el embarazo ACOG 2013
Hipertension en el embarazo ACOG 2013Hipertension en el embarazo ACOG 2013
Hipertension en el embarazo ACOG 2013
 
ENARM Obstetricia ginecologia
ENARM Obstetricia ginecologiaENARM Obstetricia ginecologia
ENARM Obstetricia ginecologia
 
PEDIATRIA ENARM
PEDIATRIA ENARMPEDIATRIA ENARM
PEDIATRIA ENARM
 
10.NEUMOLOGIA
10.NEUMOLOGIA10.NEUMOLOGIA
10.NEUMOLOGIA
 
9.ENDOCRINOLOGIA
9.ENDOCRINOLOGIA9.ENDOCRINOLOGIA
9.ENDOCRINOLOGIA
 
7.DERMATOLOGIA
7.DERMATOLOGIA7.DERMATOLOGIA
7.DERMATOLOGIA
 
6.INFECTOLOGIA
6.INFECTOLOGIA6.INFECTOLOGIA
6.INFECTOLOGIA
 
5.NEFROLOGIA
5.NEFROLOGIA5.NEFROLOGIA
5.NEFROLOGIA
 
4.GASTROENTEROLOGIA Y CIRUGIA
4.GASTROENTEROLOGIA Y CIRUGIA4.GASTROENTEROLOGIA Y CIRUGIA
4.GASTROENTEROLOGIA Y CIRUGIA
 
3.REUMATOLOGIA
3.REUMATOLOGIA3.REUMATOLOGIA
3.REUMATOLOGIA
 
2.CARDIOLOGIA
2.CARDIOLOGIA2.CARDIOLOGIA
2.CARDIOLOGIA
 
1.HEMATOLOGIA
1.HEMATOLOGIA1.HEMATOLOGIA
1.HEMATOLOGIA
 
ENARM COMPENDIO
ENARM  COMPENDIOENARM  COMPENDIO
ENARM COMPENDIO
 

Último

GENERALIDADES DEL SISTEMA HEMATOPOYETICO.pptx
GENERALIDADES DEL SISTEMA HEMATOPOYETICO.pptxGENERALIDADES DEL SISTEMA HEMATOPOYETICO.pptx
GENERALIDADES DEL SISTEMA HEMATOPOYETICO.pptxRuthHudtwalcker1
 
OXIGENO TERAPIA: AEROSOLTERAPIA EN PACIENTES
OXIGENO TERAPIA: AEROSOLTERAPIA  EN PACIENTESOXIGENO TERAPIA: AEROSOLTERAPIA  EN PACIENTES
OXIGENO TERAPIA: AEROSOLTERAPIA EN PACIENTESandrescacha
 
Flashcard Anatomía del Craneo: Neurocráneo y Vicerocráneo.
Flashcard Anatomía del Craneo: Neurocráneo y Vicerocráneo.Flashcard Anatomía del Craneo: Neurocráneo y Vicerocráneo.
Flashcard Anatomía del Craneo: Neurocráneo y Vicerocráneo.sczearielalejandroce
 
(2024-04-30). ACTUALIZACIÓN EN PREP FRENTE A VIH (PPT)
(2024-04-30). ACTUALIZACIÓN EN PREP FRENTE A VIH (PPT)(2024-04-30). ACTUALIZACIÓN EN PREP FRENTE A VIH (PPT)
(2024-04-30). ACTUALIZACIÓN EN PREP FRENTE A VIH (PPT)UDMAFyC SECTOR ZARAGOZA II
 
Histologia del sistema respiratorio y sus funciones
Histologia del sistema respiratorio y sus funcionesHistologia del sistema respiratorio y sus funciones
Histologia del sistema respiratorio y sus funcionesCarlosVazquez410328
 
DETERMINISMO DEL TRABAJO DE PARTO-1.pptx
DETERMINISMO DEL TRABAJO DE PARTO-1.pptxDETERMINISMO DEL TRABAJO DE PARTO-1.pptx
DETERMINISMO DEL TRABAJO DE PARTO-1.pptxfiorellaanayaserrano
 
SISTEMA NERVIOSO ORGANIZADOR GRAFICO.pdf
SISTEMA NERVIOSO ORGANIZADOR GRAFICO.pdfSISTEMA NERVIOSO ORGANIZADOR GRAFICO.pdf
SISTEMA NERVIOSO ORGANIZADOR GRAFICO.pdfFabiTorrico
 
FISIOLOGIA BACTERIANA y mecanismos de acción (1).pptx
FISIOLOGIA BACTERIANA y mecanismos de acción (1).pptxFISIOLOGIA BACTERIANA y mecanismos de acción (1).pptx
FISIOLOGIA BACTERIANA y mecanismos de acción (1).pptxLoydaMamaniVargas
 
Patologías de los eritrocitos-Histologia
Patologías de los eritrocitos-HistologiaPatologías de los eritrocitos-Histologia
Patologías de los eritrocitos-Histologia Estefa RM9
 
Sangrado Uterino Anormal. Dr Carlos Quiroz_052747.pptx
Sangrado Uterino Anormal. Dr Carlos Quiroz_052747.pptxSangrado Uterino Anormal. Dr Carlos Quiroz_052747.pptx
Sangrado Uterino Anormal. Dr Carlos Quiroz_052747.pptxCarlos Quiroz
 
SEGUNDA Y TERCERA SEMANA DEL DESARROLLO EMBRIONARIO.pptx
SEGUNDA  Y  TERCERA  SEMANA  DEL  DESARROLLO  EMBRIONARIO.pptxSEGUNDA  Y  TERCERA  SEMANA  DEL  DESARROLLO  EMBRIONARIO.pptx
SEGUNDA Y TERCERA SEMANA DEL DESARROLLO EMBRIONARIO.pptxArian753404
 
SISTEMA OBLIGATORIO GARANTIA DE LA CALIDAD EN SALUD SOGCS.pdf
SISTEMA OBLIGATORIO GARANTIA DE LA CALIDAD EN SALUD SOGCS.pdfSISTEMA OBLIGATORIO GARANTIA DE LA CALIDAD EN SALUD SOGCS.pdf
SISTEMA OBLIGATORIO GARANTIA DE LA CALIDAD EN SALUD SOGCS.pdfTruGaCshirley
 
PRIMEROS AUXILIOS BOMBEROS 2024 actualizado
PRIMEROS AUXILIOS BOMBEROS 2024 actualizadoPRIMEROS AUXILIOS BOMBEROS 2024 actualizado
PRIMEROS AUXILIOS BOMBEROS 2024 actualizadoNestorCardona13
 
infografía seminario.pdf.................
infografía seminario.pdf.................infografía seminario.pdf.................
infografía seminario.pdf.................ScarletMedina4
 
1. PRESENTACION DE MANEJO DE CLAVE ROJA
1. PRESENTACION DE  MANEJO DE CLAVE ROJA1. PRESENTACION DE  MANEJO DE CLAVE ROJA
1. PRESENTACION DE MANEJO DE CLAVE ROJAanamamani2023
 
ESQUEMA DE VACUNACION Y CADENA DE FRIO.pptx
ESQUEMA DE VACUNACION Y CADENA DE FRIO.pptxESQUEMA DE VACUNACION Y CADENA DE FRIO.pptx
ESQUEMA DE VACUNACION Y CADENA DE FRIO.pptxCinthiaPQuimis
 
1 mapa mental acerca del virus VIH o sida
1 mapa mental acerca del virus VIH o sida1 mapa mental acerca del virus VIH o sida
1 mapa mental acerca del virus VIH o sidagsandovalariana
 
ACRONIMO TIMERS TRATAMIENTO DE HERIDAS AVANZADAS
ACRONIMO TIMERS TRATAMIENTO DE HERIDAS AVANZADASACRONIMO TIMERS TRATAMIENTO DE HERIDAS AVANZADAS
ACRONIMO TIMERS TRATAMIENTO DE HERIDAS AVANZADASjuanjosenajerasanche
 
CLASE DE VIH/ETS - UNAN 2024 PEDIATRIA I
CLASE DE VIH/ETS - UNAN 2024 PEDIATRIA ICLASE DE VIH/ETS - UNAN 2024 PEDIATRIA I
CLASE DE VIH/ETS - UNAN 2024 PEDIATRIA ILucy López
 
CLASE 2 de mecanismo de agresión y defensa I
CLASE 2 de mecanismo de agresión y defensa ICLASE 2 de mecanismo de agresión y defensa I
CLASE 2 de mecanismo de agresión y defensa IAnaB593936
 

Último (20)

GENERALIDADES DEL SISTEMA HEMATOPOYETICO.pptx
GENERALIDADES DEL SISTEMA HEMATOPOYETICO.pptxGENERALIDADES DEL SISTEMA HEMATOPOYETICO.pptx
GENERALIDADES DEL SISTEMA HEMATOPOYETICO.pptx
 
OXIGENO TERAPIA: AEROSOLTERAPIA EN PACIENTES
OXIGENO TERAPIA: AEROSOLTERAPIA  EN PACIENTESOXIGENO TERAPIA: AEROSOLTERAPIA  EN PACIENTES
OXIGENO TERAPIA: AEROSOLTERAPIA EN PACIENTES
 
Flashcard Anatomía del Craneo: Neurocráneo y Vicerocráneo.
Flashcard Anatomía del Craneo: Neurocráneo y Vicerocráneo.Flashcard Anatomía del Craneo: Neurocráneo y Vicerocráneo.
Flashcard Anatomía del Craneo: Neurocráneo y Vicerocráneo.
 
(2024-04-30). ACTUALIZACIÓN EN PREP FRENTE A VIH (PPT)
(2024-04-30). ACTUALIZACIÓN EN PREP FRENTE A VIH (PPT)(2024-04-30). ACTUALIZACIÓN EN PREP FRENTE A VIH (PPT)
(2024-04-30). ACTUALIZACIÓN EN PREP FRENTE A VIH (PPT)
 
Histologia del sistema respiratorio y sus funciones
Histologia del sistema respiratorio y sus funcionesHistologia del sistema respiratorio y sus funciones
Histologia del sistema respiratorio y sus funciones
 
DETERMINISMO DEL TRABAJO DE PARTO-1.pptx
DETERMINISMO DEL TRABAJO DE PARTO-1.pptxDETERMINISMO DEL TRABAJO DE PARTO-1.pptx
DETERMINISMO DEL TRABAJO DE PARTO-1.pptx
 
SISTEMA NERVIOSO ORGANIZADOR GRAFICO.pdf
SISTEMA NERVIOSO ORGANIZADOR GRAFICO.pdfSISTEMA NERVIOSO ORGANIZADOR GRAFICO.pdf
SISTEMA NERVIOSO ORGANIZADOR GRAFICO.pdf
 
FISIOLOGIA BACTERIANA y mecanismos de acción (1).pptx
FISIOLOGIA BACTERIANA y mecanismos de acción (1).pptxFISIOLOGIA BACTERIANA y mecanismos de acción (1).pptx
FISIOLOGIA BACTERIANA y mecanismos de acción (1).pptx
 
Patologías de los eritrocitos-Histologia
Patologías de los eritrocitos-HistologiaPatologías de los eritrocitos-Histologia
Patologías de los eritrocitos-Histologia
 
Sangrado Uterino Anormal. Dr Carlos Quiroz_052747.pptx
Sangrado Uterino Anormal. Dr Carlos Quiroz_052747.pptxSangrado Uterino Anormal. Dr Carlos Quiroz_052747.pptx
Sangrado Uterino Anormal. Dr Carlos Quiroz_052747.pptx
 
SEGUNDA Y TERCERA SEMANA DEL DESARROLLO EMBRIONARIO.pptx
SEGUNDA  Y  TERCERA  SEMANA  DEL  DESARROLLO  EMBRIONARIO.pptxSEGUNDA  Y  TERCERA  SEMANA  DEL  DESARROLLO  EMBRIONARIO.pptx
SEGUNDA Y TERCERA SEMANA DEL DESARROLLO EMBRIONARIO.pptx
 
SISTEMA OBLIGATORIO GARANTIA DE LA CALIDAD EN SALUD SOGCS.pdf
SISTEMA OBLIGATORIO GARANTIA DE LA CALIDAD EN SALUD SOGCS.pdfSISTEMA OBLIGATORIO GARANTIA DE LA CALIDAD EN SALUD SOGCS.pdf
SISTEMA OBLIGATORIO GARANTIA DE LA CALIDAD EN SALUD SOGCS.pdf
 
PRIMEROS AUXILIOS BOMBEROS 2024 actualizado
PRIMEROS AUXILIOS BOMBEROS 2024 actualizadoPRIMEROS AUXILIOS BOMBEROS 2024 actualizado
PRIMEROS AUXILIOS BOMBEROS 2024 actualizado
 
infografía seminario.pdf.................
infografía seminario.pdf.................infografía seminario.pdf.................
infografía seminario.pdf.................
 
1. PRESENTACION DE MANEJO DE CLAVE ROJA
1. PRESENTACION DE  MANEJO DE CLAVE ROJA1. PRESENTACION DE  MANEJO DE CLAVE ROJA
1. PRESENTACION DE MANEJO DE CLAVE ROJA
 
ESQUEMA DE VACUNACION Y CADENA DE FRIO.pptx
ESQUEMA DE VACUNACION Y CADENA DE FRIO.pptxESQUEMA DE VACUNACION Y CADENA DE FRIO.pptx
ESQUEMA DE VACUNACION Y CADENA DE FRIO.pptx
 
1 mapa mental acerca del virus VIH o sida
1 mapa mental acerca del virus VIH o sida1 mapa mental acerca del virus VIH o sida
1 mapa mental acerca del virus VIH o sida
 
ACRONIMO TIMERS TRATAMIENTO DE HERIDAS AVANZADAS
ACRONIMO TIMERS TRATAMIENTO DE HERIDAS AVANZADASACRONIMO TIMERS TRATAMIENTO DE HERIDAS AVANZADAS
ACRONIMO TIMERS TRATAMIENTO DE HERIDAS AVANZADAS
 
CLASE DE VIH/ETS - UNAN 2024 PEDIATRIA I
CLASE DE VIH/ETS - UNAN 2024 PEDIATRIA ICLASE DE VIH/ETS - UNAN 2024 PEDIATRIA I
CLASE DE VIH/ETS - UNAN 2024 PEDIATRIA I
 
CLASE 2 de mecanismo de agresión y defensa I
CLASE 2 de mecanismo de agresión y defensa ICLASE 2 de mecanismo de agresión y defensa I
CLASE 2 de mecanismo de agresión y defensa I
 

8.NEUROLOGIA

  • 2.
  • 3.
  • 4. MANIFESTACIONES DE LESION AL SNC • Como se muestra una lesión del TRACTO ÓPTICO? • R = Como CUADRANTOPSIA homónima contralateral. • Que caracteriza al síndrome de Horner? • R = Miosis, ptosis y anhidrosis debido a lesión de las vías del SISTEMA SIMPÁTICO ispolaterales. 1) Miosis: Se debe a denervación simpática del musculo tarsal 2) Anhidrosis: Consecuencia de la denervación simpática del musculo tarsal 3) Para localizar la lesión se realizan pruebas con cocaína, hidroxifentamina y fenilefrina. • Que ocasiona las pupilas de Argyll Robertson? 1) Son PUPILAS IRREGULARES Y MÁS PEQUEÑAS de lo normal en la oscuridad. 2) Demuestran falta del reflejo fotomotor, tanto directo como consensual, sin embargo conservan la acomodación. 3) Se asocia a NEUROSIFILIS. • Que ocasiona las pupilas de Holmes-Adie? 1) Se trata de una PUPILA GRANDE Y TÓNICA de forma irregular 2) Tiene reacción pobre ante estimulo luminoso 3) Suele indicar una lesión de las vías parasimpáticas posganglionares, a nivel del ganglio ciliar o los nervios ciliares cortos. 4) Se ve una REACCIÓN CONSTRICTORA EXAGERADA ante la administración de agonistas muscarinicos como la meticolina o la PILOCARPINA. 5) Se asocian a reflejos de estiramiento muscular disminuidos o ausentes.
  • 5. MANIFESTACIONES DE LESION AL SNC • Cuales son las manifestaciones oculares en caso de obstrucción de las RADIACIONES ÓPTICAS? • R = CUADRANTOPSIAS HOMONIMAS • -A 75-year-old woman presents with symptoms of visual change and facial weakness. On examination, the pupils are equal and reactive to light, the fundi appear normal, and there is a right homonymous visual field defect. Which of the following is the most likely cause of the right homonymous hemianopia? • (A) right optic nerve • (B) chiasm • (C) right optic radiations • (D) right occipital lobe • (E) left optic radiations • - The hemianopia is due to a lesion of the left optic radiations. The posterior cerebral artery arises from the basilar artery but is sometimes a branch of the internal carotid. With posterior cerebral artery lesions affecting the occipital cortex, it is possible for the hemianopia to be an isolated finding.
  • 6.
  • 7. • CORRELACIÓN TOPOGRÁFICA DE DEFECTOS DEL CAMPO VISUAL. A: abolición CV izquierdo (lesión nervio óptico izquierdo). B: escotoma yuncional. C: hemianopsia bitemporal heterónima (lesión quiasmática). D: hemianopsia homónima derecha (lesión quiasmática). E: sectoronopsia cuádruple homónima derecha (lesión ganglio geniculado izquierdo). F: Sectoranopsia horizontal derecha (lesión ganglio geniculado izquierdo). G: Cuadrantanopsia homónima inferior derecha (lesión lóbulo parietal izquierdo). H: Cuadrantanopsia homónima superior derecha (lesión lóbulo temporal izquierdo). I: Hemianopsia homónima derecha con respeto macular (lesión occipital izquierda).
  • 8.
  • 9. MANIFESTACIONES DE LESION AL SNC • Que es la disautonomia? • R = Proceso patológico central o periférico que se manifiesta por anomalías de la regulación de la TA, diaforesis termorreguladora, función sexual. • Que es el mioclono? 1) Es común en epilepsia, presentando ESPASMOS. Se presenta en casos de anoxia, hiperuricemia. 2) El tratamiento es con levodopa, privación alcohólica y responde a acido valproico y benzodiacepinas. • Que motoneurona se ve afectada en la espasticidad? • R = MOTONEURONA SUPERIOR • Cual es tratamiento de espasticidad? • R = Fisioterapia, dantroleno debilita la contractura muscular al interferir con el calcio, diazepam, inyección de fenol para disminuir la espasticidad selectiva o toxina botulínica.
  • 10. MANIFESTACIONES DE LESION AL SNC • En que zona se encuentra la lesión en un paciente estuporoso/comatoso en ausencia unilateral de respuesta a pesar de la aplicación de estímulos en ambos lados del cuerpo? • R = CORTICOESPINAL • En que zona se encuentra la lesión en paciente en ausencia bilateral de respuesta a estímulos? • R = Afección de TALLO ENCEFÁLICO, lesión bilateral de vía piramidal • La postura de decorticacion a que daño se debe? • R = Lesión de CAPSULA INTERNA y del PEDÚNCULO CEREBRAL ANTERIOR. • La postura de decerebracion a que daño se debe? • R = DESTRUCCIÓN MESENCÉFALO y de la porción anterior del puente de vacilio • Que causa el síndrome de encerramiento o estado desaferentado? 1) Lesiones agudas que afectan principalmente la PARTE ANTERIOR DEL PUENTE DE VACILIO Y RESPETAN EL TEGUMENTO. 2) Tienen respuesta ocular y están consientes pero en estado comatoso y se RECUPERAN DE 2-3ª.
  • 12.
  • 13.
  • 14. MOTONEURONAS • Que caracteriza a las lesiones de la motoneurona? • R = Debilidad sin perdida sensorial • Cual es la clasificación de la enfermedad de motoneurona y que caracteriza a cada una de ellas? 1) Parálisis bulbar progresiva: degeneración que predomina en núcleos motores de pares craneales 2) Parálisis pseudobulbar: se debe a enfermedad cortico bulbar alterando neurona motora superior 3) Atrofia muscular espinal progresiva: Hay degeneración de las astas anteriores de la medula espinal afectando a neurona motora inferior 4) Esclerosis lateral primaria: Déficit puro de neurona motora superior en extremidades inferiores 5) Esclerosis lateral aminotrofica: Déficit mixto de NMS e INFERIOR en extremidades pudiendo alterar el estado cognitivo con DEMENCIA FRONTOTEMPORAL O PARKINSONISMO. • Cual es el cuadro clínico de enfermedad degenerativa de neuronas motoras? 1) Alteración bulbar: dificultad para la deglución, masticación, tos, respiración y disartria. 2) Parálisis bulbar progresiva: descanso del paladar disminuyendo reflejo nauseoso, tos débil y desgaste muscular. Lingual con fasciculaciones. 3) Parálisis pseudobulbar: Contractura y espasmo lingual, disminución de la motilidad de la lengua. Hay trastorno motor en extremidades con esfínteres normales 4) ESCLEROSIS LATERAL AMINOTROFICA: ES LETAL de 3-5 años comúnmente POR INFECCIÓN PULMONAR, presentándose con periodos de REMISIÓN Y EXACERBACIÓN DE DIFICULTAD PARA CAMINAR, TRAGAR , TORPEZA Y FINALMENTE RESPIRAR CON CARÁCTER AUTOSOMICO DOMINANTE EN GEN 21.
  • 15. MOTONEURONAS • Cuales son los signos positivos y negativos que indican lesión a la MOTONEURONA SUPERIOR o síndrome piramidal? 1) Positivos: ESPASTICIDAD, HIPERREFLEXIA en reflejos de estiramiento muscular, clonus, distonía, atetosis y exaltación de reflejos cutáneos como signo de Babinski 2) Negativos: Debilidad o paresia hemicorporal y pérdida de la destreza, coordinación y control motor. 3) La combinación de ambos afectan la ejecución normal del movimiento con un deterioro en la calidad de vida del paciente. • Cual es el cuadro clínico del síndrome de la MOTONEURONA INFERIOR? • R = ATROFIA muscular por denervación, parálisis o PARESIA FLÁCIDA de todos los músculos de la unidad motora afectada, HIPOTONÍA, hiporreflexia o ARREFLEXIA al interrumpirse la parte eferente del arco reflejo y fasciculaciones producidas por la actividad espontanea de la unidad motora. • Cuales son las características clínicas del síndrome de la columna posterior? 1) Se caracteriza por perdida o DISMINUCIÓN DE LA SENSACIÓN DE VIBRACIÓN, SENSACIÓN DE POSICIÓN, DISCRIMINACIÓN DE DOS PUNTOS, TACTO Y RECONOCIMIENTO DE LA FORMA 2) Estimulos que no eran dolorosos SE CONVIERTEN EN DOLOROSOS y LOS ESTÍMULOS DOLOROSOS SE DESENCADENAN CON UMBRALES DE ESTIMULACIÓN MAS BAJOS. • Que caracteriza al síndrome de la arteria espinal anterior? • R = También conocido como síndrome medular ventral que ocasiona PARAPLEJIA O CUADRIPLEJIA, perdida bilateral de la sensación de dolor y temperatura por debajo del nivel de la lesión. • Que estudios realizas para diagnosticar la enfermedad de la motoneurona? • R = ELECTROMIOGRAFÍA: muestra DENERVACIÓN, EN ESCLEROSIS LATERAL AMINOTROFICA debe presentarse en 3 extremidades mínimo. LINFOMA DE HODKING puede cursar con NEUROPATÍA MOTORA. • Cual es el tratamiento de lesiones de neuronas motoras? • R = RICUZOL: disminuye la LIBERACIÓN PRESINAPTICA DE GLUTAMATO haciendo lenta la ELA progresiva. PLASMAFERESIS logra mejoría. ANTICOLINERGICOS: atropina y amitriptilina
  • 16. MOTONEURONAS • - A 53-year-old man complains of clumsiness with both hands, like having difficulty doing up buttons or using his keys. Physical examination reveals fasciculations of his thigh and forearm muscles; diffuse muscle weakness, loss of muscle bulk, and increased tone in the upper and lower limbs. There is generalized hyperreflexia, and positive Babinski signs bilaterally. Which of the following is the most likely natural progression of this condition? • (A) a long history of remissions and exacerbations (sclerosis lateral aminotrofica) • (B) sensory loss in the distribution of peripheral nerves • (C) focal seizures • (D) a progressively downhill course • (E) cogwheel rigidity • - This man has amyotrophic lateral sclerosis (ALS). The disease causes neuronal loss in the anterior horns of the spinal cord and motor nuclei of the lower brain stem. The disease is one of constant progression, rather than remissions and exacerbations, and death usually occurs within 5 years. There is no sensory loss and no seizure diathesis, because only the motor system is involved. There can be signs ofhyperreflexia and spasticity, depending on the balance of upper and lower motor neuron damage, but not cogwheel rigidity. • -A 45-year-old man presents with weakness and fasciculations in his arms and legs. His cranial nerves are normal, but there is weakness of his left handgrip and right leg quadriceps with loss of muscle bulk. There are obvious fasciculations over the left forearm and right thigh. Tone is increased in the arms and legs and the reflexes are brisk. Which of the following is the most likely diagnosis? • (A) amyotrophic lateral sclerosis (ALS) • (B) myotonic muscular dystrophy • (C) amyotonia congenita • (D) tabes dorsalis • (E) migraine • - The most common initial symptom of ALS is weakness and wasting of the extremities. The fasciculations can be a very prominent part of the disease. This is rare in other neurologic disorders.
  • 17.
  • 18.
  • 19. TRAUMATISMO • Que te indica el signo de mapache? • R = Equimosis en parpados debido a fractura de base del craneo por TCE • Que maniobra resulta útil en el TCE que causa salida de LCR por nariz u oídos (contiene glucosa)? • R = CONSERVADOR, con ELEVACIÓN DE LA CABEZA, RESTRICCIÓN DE LÍQUIDOS, administración de acetazolamida EN CASO DE PERSISTENCIA SE REALIZA DRENAJE LUMBAR SUBARACNOIDEO • Cual es el cuadro clínico de traumatismo en medula espinal? 1) La transección medular completa resulta en PARÁLISIS FLÁCIDA INMEDIATA Y PERDIDA DE LA SENSIBILIDAD DEBAJO DE LA LESIÓN. 2) Se pierde retención urinaria y fecal. 3) Al final hay paraplejia en las piernas en flexión o extensión. 4) En las LESIONES MENORES los pacientes que dan con DEBILIDAD LEVE DE LAS EXTREMIDADES, TRASTORNOS SENSITIVOS DISTALES O AMBOS. • Que caracteriza al síndrome de BROWN-SEQUARD? • R = Es una LESIÓN UNILATERAL de la MEDULA ESPINAL que da lugar a un TRASTORNO MOTOR IPSOLATERAL con deterioro concomitante de la PROPIOSEPCION y PERDIDA CONTRALATERAL de las apreciaciones del DOLOR Y TEMPERATURA • Cual es el manejo del traumatismo medular? • R = INMOBILIDAD. GRANDES DOSIS DE ESTEROIDES metilprednisolona 300 mg en bolo seguidos por 5.4 mg/kg/hr x 24 hrs.
  • 20. TRAUMATISMO • Cual es la principal etiología de un HEMATOMA SUBDURAL y sus principales características clínicas? • R = A subdural hematoma is almost always of VENOUS ORIGIN and secondary to a minor or severe INJURY TO THE HEAD, but may occur in blood dyscrasias or cachexia in the absence of trauma. ACUTE subdural hematomas commonly present with a FLUCTUATING LEVEL OF CONSCIOUSNESS and significant cerebral damage. CHRONIC subdurals may also present with SEIZURES OR PAPILLEDEMA • -A 94-year-old man presents with progressive headaches, light-headedness, drowsiness, and unsteady gait over 6 weeks. On examination, his blood pressure is 160/90 mm Hg, pulse 70/min, lungs clear, and he has no focal weakness. His gait is unsteady but sensation in the feet is normal. A CT scan reveals a hyperintense clot over the left cerebral cortex. Which of the following is the most likely cause for this clot? • (A) is venous in origin • (B) is arterial in origin • (C) is from injury to the middle meningeal artery • (D) is from a subarachnoid hemorrhage • (E) is from injury to the middle cerebral artery
  • 21.
  • 22.
  • 23.
  • 24.
  • 25. ESCALA DE GLASGOW MODIFICADA (NIÑOS) PRUEBA RESPUESTA PUNTUACIÓN APERTURA DE OJOS Espontanea A órdenes Al estimulo doloroso Nula 4 3 2 1 LLANTO COMO RESPUESTA VERBAL  Palabras apropiadas y sonrisas, fija la mirada y sigue los objetos.  Tiene llanto, pero consolable.  Persistente e irritable  Agitado.  Sin respuesta. 5 4 3 2 1 RESPUESTA MOTORA Obedece ordenes Localiza el dolor Retirada ante el dolor Flexión inapropiada Extensión Nula 6 5 4 3 2 1
  • 26. TCE Leve .... 13-15 puntos TCE Moderado.. 9-12 puntos TCE Severo.... 8 puntos o menos (requiere intubación)
  • 27. Si EGC: 15-----------------------Consciente Si EGC: 13-14-------------------Estupor ligero Si EGC: 11-12-------------------Estupor moderado Si EGC: 9-10--------------------Estupor profundo Si EGC: 7-8---------------------Coma superficial Si EGC: 5-6---------------------Coma moderado Si EGC: 3-4---------------------Coma profundo
  • 28. Según el tamaño Mioticas  Diámetro < 2mm Medias  Diámetro 2-5mm Midriaticas  Diámetro > 5mm Según relación entre ellas Isocoricas  Iguales Anisocoricas  Desiguales Discoricas Forma irregular Reactivas Contracción al foco luminoso Según respuesta a la luz Arreactivas Inmóviles al foco luminoso REACTIVIDAD PUPILAR.
  • 30. SIRINGOMIELIA • Que es la siringomielia? • R = CAVITACIÓN DE LA MEDULA ESPINAL por DESTRUCCIÓN O DEGENERACIÓN DE LA SUSTANCIA GRIS O BLANCA asociada a la malformación de ARNOLD CHIARI. Cuenta con 2 etiologías que son la CONGÉNITA Y ADQUIRIDA. Presenta un síndrome siringomielico que consiste en: 1) Hipoestesia termoalgesica suspendida disociada 2) PERDIDA DE FUERZA Y AMIOTROFIA 3) ARREFLEXIA 4) Trastornos vegetativos y tróficos 5) Perdida de sensibilidad vibratoria y posicional
  • 31. SÍNDROME DEL TÚNEL CARPIANO • A que se asocia el síndrome del túnel carpiano? • R = HIPOTIROIDISMO, EMBARAZO, diabetes, AR o ACROMEGALIA. • Cuales son las manifestaciones clínicas del síndrome del túnel carpiano? 1) HORMIGUEO, dolor, ENTUMECIMIENTO EN LA MANO, sobre todo al comienzo de la mañana y en ocasiones con DEBILIDAD DE LA MUSCULATURA TENAR con atrofia del abductor corto del pulgar. 2) Puede observarse el SIGNO DEL TINEL que se provoca con un golpesito en el túnel carpiano REPRODUCIENDO DOLOR Y HORMIGUEO. • Como dx el sx del túnel carpiano? • R = CLÍNICA + ELECTROMIOGRAFÍA. • Cual es el manejo del sx del túnel carpiano? • R = Descompresión quirúrgica.
  • 32. COMPRESIÓN DEL NERVIO CUBITAL • Como se manifiesta clínicamente la compresión del nervio cubital? 1) ADORMECIMIENTO EN EL QUINTO DEDO y en el lado MEDIAL de la mano. 2) El dolor y las parestesias pueden iniciar en el codo e irradiarse a la mano. 3) LAS PARESTESIAS SE EXACERBAN CON LA FLEXIÓN DEL CODO. • Como diagnosticas la compresión del nervio cubital? • R = Electrofisiología. • Cual es el manejo de la compresión del nervio cubital? • R = Los ESTEROIDES Y LOS AINES. Se recomienda cirugía.
  • 33. NERVIO CUBITAL • Cual es la sintomatología en caso de daño al nervio cubital? • R = Injury to the ulnar nerve results in impaired adduction and abduction of the fingers. The nerve is commonly injured in ELBOW DISLOCATIONS and fractures. The fibers arise from the eighth cervical and the first thoracic segments. The ulnar is a mixed nerve with sensory supply to the medial hand. • - A 40-year-old man is injured in a car accident and fractures his left elbow. He now complains of numbness of his fourth and fifth fingers, and weakness in his hand grip. Neurologic findings confirm weakness of handgrip with weakness of finger abduction and adduction, and decreased sensation over the fifth finger and lateral aspect of fourth finger. Which of the following is the most likely diagnosis? • (A) ulnar nerve injury • (B) radial nerve injury • (C) median nerve injury • (D) carpal tunnel syndrome • (E) axillary nerve injury
  • 34. COMPRESIÓN DEL NERVIO RADIAL • Como se manifiesta la compresión del nervio radial? 1) “Parálisis de la noche del sábado” por dejar el brazo colgando después de una borrachera. 2) MUÑECA FLÁCIDA y debilidad a la extensión de los dedos.
  • 35. CIATICO Y TARSO • Cual es la etiología mas común de la parálisis del ciático? • R = INYECCIONES INTRAMUSCULARES profundas • Que sintomatología da el síndrome del túnel del tarso? • R = NERVIO TIBIAL, rama del ciático, cuando se comprime da origen a DOLOR, PARESTESIAS Y ENTUMECIMIENTO SOBRE BASE DEL PIE PREDOMINANDO DURANTE LA NOCHE sin afectar talón.
  • 36. MERALGIA PARESTESICA • Que causa y como se manifiesta clínicamente la meralgia parestesica? 1) Se debe al atrapamiento del NERVIO CUTÁNEO LATERAL DEL MUSLO POR DEBAJO DEL LIGAMENTO INGUINAL. 2) Se produce DOLOR URENTE, PARESTESIAS Y ENTUMECIMIENTO DE LA CARA ANTEROLATERAL DEL MUSLO. 3) Se presenta en pacientes con EXCESO DE PESO por lo que el TRATAMIENTO ES BAJAR DE PESO. 4) Nervio FEMOROCUTANEO EXTERNO, se origina en RAÍCES L2 Y L3 EL CUAL SE PRESIONA EN CASO DE EMBARAZO, DIABETES Y OBSESOS.
  • 37. LESIONES DE LOS PARES CRANEALES • Que datos clínicos se encuentran en el daño al III (OCULOMOTOR) PAR CRANEAL? 1) Frecuentemente causa ANISOCORIA, en la que la pupila mas dilatada denota al LADO AFECTADO. 2) El fenómeno de MARCUS GUNN consiste en que con el ESTIMULO LUMINOSO NO OBTENEMOS RESPUESTA EN EL LADO AFECTADO después de presentar contracción por efecto de reflejo consensual al estimular contralateralmente 3) Suele deberse a CAUSAS HIPOXICAS 4) En pacientes con compresión cerebral se mostrara midriasis ipsolateral al sitio afectado como resultado de compresión del III par craneal en cualquier parte de su recorrido debido a que las fibras que inervan el esfínter de la pupila discurren en porción periférica del nervio. 5) PTOSIS PALPEBRAL CONTRALATERAL a la lesión. 6) La lesión en este par craneal afecta los MÚSCULOS ELEVADOR DEL PARPADO, recto medial, recto superior, recto inferior y oblicuo inferior (TODOS MENOS EL RECTO EXTERNO Y EL OBLICUO SUPERIOR), lo que provoca que los movimientos de ADUCCIÓN, ELEVACIÓN Y DEPRESIÓN DEL OJO IPSOLATERAL ESTÉN AFECTADOS. 7) Lo anterior provoca que cuando el paciente este de frente el ojo tiende a DESVIARSE HACIA ABAJO Y AFUERA. • Que datos clínicos se encuentran en el daño al IV (PATÉTICO O TROCLEAR) PAR CRANEAL? 1) Presentan debilidad del MUSCULO OBLICUO SUPERIOR 2) DIPLOPIA CON ADUCCION Y HACIA ABAJO • Que datos clínicos se encuentran en el daño al VI (OCULOMOTOR EXTERNO) PAR CRANEAL? 1) Afecta al MUSCULO RECTO LATERAL 2) El ojo afectado se dirige A LA LINEA MEDIA CON INCAPACIDAD DE ABDUCIR EL OJO.
  • 38. LESIONES DE LOS PARES CRANEALES • Cual es el cuadro clínico de la parálisis del 3er par craneal ? • R = Third nerve palsy can result in PTOSIS of the eyelid. There is also loss of the ability to open the eye, and the EYEBALL IS DEVIATED OUTWARD AND SLIGHTLY DOWNWARD. With complete lesions, THE PUPIL IS DILATED, DOES NOT REACT TO LIGHT, and loses the power of accommodation. In diabetes, the pupil is often spared. The sixth cranial nerve can also be affected by diabetes, but this is much less common. • -A 60-year-old man with diabetes acutely develops double vision and discomfort in his left eye. On examination, there is ptosis of the left eyelid, the eye is rotated down and out, and the pupil is 3 mm and reactive to light. The right eye is normal. Which of the following is the most likely diagnosis? • (A) fourth nerve palsy • (B) diabetic autonomic neuropathy • (C) third nerve palsy • (D) sixth nerve palsy • (E) seventh nerve palsy
  • 39. LESIONES DE LOS PARES CRANEALES • Que datos clínicos se encuentran en el daño al V (TRIGÉMINO) PAR CRANEAL? 1) La parálisis de la primera división (OFTÁLMICA) involucra PERDIDA DE LA SENSIBILIDAD EN LA FRENTE junto con parálisis del III par y IV. 2) La parálisis de la segunda división (MAXILAR) resulta en la PERDIDA DE SENSIBILIDAD DE LA MEJILLA y se debe a lesiones del seno cavernoso. 3) La parálisis de la tercera división (MANDIBULAR) resulta en DÉFICIT EN LA MASTICACIÓN • Que cuadro clínico resulta del síndrome ocasionado por la afección del ganglio de Gasser? • R = Síndrome caracterizado por DOLOR FACIAL, CEFALEA, PERDIDA DE LA SENSIBILIDAD SENSORIAL, SORDERA Y PARÁLISIS DEL VI Y VII PARES. EL DOLOR AUMENTA AL MOVER LA MANDÍBULA. • Cual es el cuadro clínico resultante de la neuralgia del glosofaríngeo? • R = Es un trastorno unilateral que afecta principalmente a V3 presentándose en forma de ESPASMOS DOLOROSOS que pueden durar de segundos a minutos y pueden estar asociados a estímulos cutáneos y auditivos. • Cual es la etiología de la parálisis de Bell? • R = Se asocia a HERPES VIRUS • Que características clínicas tiene la parálisis de Bell? 1) PARÁLISIS DEL VII PAR CON INICIO SÚBITO DE DEBILIDAD FACIAL y pérdida del reflejo acústico. 2) Hay PRÓDROMO VIRAL y adormecimiento o dolor en el oído, lengua o cara. 3) La CUERDA DEL TÍMPANO SE VE ERITEMATOSA y HAY REDUCCIÓN EN LA LAGRIMACIÓN IPSOLATERAL o salivación. 4) El paciente no puede cerrar los ojos por lo que se debe evitar el daño a la cornea 5) Se diagnostica con ELECTROMIOGRAFÍA denotándose una DISMINUCIÓN EN LA VELOCIDAD DE CONDUCCIÓN
  • 40. LESIONES DE LOS PARES CRANEALES • Cual es el manejo de la parálisis de Bell? • R = VALACICLOVIR o famciclovir y PREDNISONA • Cual es la característica clínica de la afectación del VIII (VESTIBULOCOCLEAR) par craneal? 1) Porción coclear: El principal síntoma asociado a la lesión o atrofia de esta porción es la HIPOACUSIA SENSORINEURAL. Se realiza el diagnostico por AUDIOMETRÍA. 2) Porción vestibular: VÉRTIGO POSTURAL PAROXÍSTICO BENIGNO. • Que características clínicas se observan por daño al XI (ESPINAL ACCESORIO) par craneal? • R = Dolor significativo acompañado de una DISMINUCIÓN DE LA MOVILIDAD DEL HOMBRO, siendo la lesión iatrogena la mas común (p.e la BIOPSIA DE LOS GANGLIOS LINFÁTICOS). • Cuales son los datos clínicos que involucra una lesión en los pares craneales IX Y X (GLOSOFARÍNGEO Y VAGO)? 1) DISFAGIA, parálisis de la cuerda vocal ipsolateral, así como DEBILIDAD DEL PALADAR Y LA FARINGE. 2) La ELEVACIÓN DEL PALADAR ES ASIMÉTRICA y la ÚVULA SE DESVÍA AL LADO CONTRARIO DE LA LESIÓN. 3) Se pierde la capacidad sensorial de UN TERCIO POSTERIOR DE LA LENGUA, con disminución en la sensibilidad en la faringe posterior y laringe. 4) La LESIÓN más común del VAGO es la que involucra al NERVIO LARÍNGEO RECURRENTE, lo que provoca PARÁLISIS DE LA CUERDA VOCAL IPSOLATERAL Y SUBSECUENTE DISFONÍA. 5) La neuralgia del glosofaríngeo tiene CARACTERÍSTICAS SIMILARES A LA NEURALGIA DEL TRIGÉMINO, pero es MAS EVIDENTE EN LA LENGUA, LAS AMÍGDALAS, OÍDO Y ANGULO DE LA MANDIBULA
  • 41. LESIONES DE LOS PARES CRANEALES • Que características clínicas tiene el daño al XII (HIPOGLOSO) par craneal? • R = DEBILIDAD en la lengua, FASCICULACIONES y DESVIACIÓN DE LA LENGUA hacia el lado de la lesión. • Que medicamento es de elección en neuralgia del trigémino y del glosofaríngeo? • R = CARBAMACEPINA, pero también puede incluir baclofen y fenitoina. • - A 52-year-old man complains of episodes of severe unilateral, stabbing facial pain that is intermittent for several hours, and then disappears for several days. The pain is describes as “electric shock-like” and only lasts a few seconds. Physical examination of the face and mouth is entirely normal. Which of the following treatments is most effective for this condition? • (A) morphine • (B) indomethacin • (C) cimetidine • (D) carbamazepine • (E) lidocaine (Xylocaine) gel • - This patient has trigeminal neuralgia. Carbamazepine (an anticonvulsant drug) is given in doses varying from 600 to 1200 mg/day. Phenytoin has also been used. The two drugs can also be used in combination. Operative procedures include alcohol injection of the nerve or ganglion, partial section of the nerve in the middle or posterior fossa, decompression of the root, and medullary tractotomy.Radiofrequency surgery can destroy pain fibers but spare motor fibers.
  • 42.
  • 43. PARALISIS DE BELL • En que consiste la parálisis de Bell? • R = Hay PARESIA IDIOPÁTICA DEL FACIAL tipo motoneurona inferior por inflamación del mismo con probable ETIOLOGÍA VIRAL COMO HERPES • Cual es el cuadro clínico de la parálisis de Bell? 1) Inicio súbito, empeora con el tiempo, EL DOLOR DEL OÍDO PRECEDE O ACOMPAÑA A LA DEBILIDAD. 2) CARA RÍGIDA JALADA HACIA UN LADO, RESTRICCIÓN UNILATERAL PARA CERRAR LOS OJOS con dificultad para comer. • Cual es el tratamiento de la parálisis de Bell? • R = 60% SIN TRATAMIENTO, 10% PERMANENTE. PREDNISONA + ACICLOVIR, se diagnostica con ELECTROMIOGRAFÍA CON DISMINUCIÓN DE LA CONDUCCIÓN.
  • 44. VERTIGO • Cual es la diferencia entre vértigo de origen central y de origen periférico? • R = In CENTRAL VERTIGO, the vertigo can be MILD AND CHRONIC. In PERIPHERAL disease, THE SYMPTOMS ARE GENERALLY MORE SEVERE, BUT FINITE (although often recurrent). • - A 63-year-old woman develops symptoms of nausea, vomiting, and dizziness, which she describes as a to-and- fro movement of the room like as if she is on a boat. Which of the following findings suggests the vertigo is central in origin? • (A) deafness • (B) symptoms are more protracted but less severe • (C) unidirectional nystagmus • (D) visual fixation inhibits vertigo and nystagmus • (E) spinning sensation is toward the fast phase of nystagmus • Cuales son las características del vértigo de ORIGEN PERIFÉRICO? • R = TINNITUS AND DEAFNESS may be found in PERIPHERAL VERTIGO, but NOT CENTRAL. The nystagmus is usually unidirectional and is never vertical. VISUAL FIXATION INHIBITS VERTIGO AND NYSTAGMUS during testing in peripheral vertigo • - A 47-year-old man presents to the emergency room with symptoms of dizziness and difficulty walking. He describes his dizziness as a spinning sensation of the room with associated nausea and vomiting. Which of the following findings suggests the vertigo is peripheral in origin? • (A) optic neuritis • (B) tinnitus • (C) bidirectional nystagmus • (D) vertical nystagmus • (E) visual fixation does not affect vertigo or nystagmus
  • 45. VERTIGO • Cual es la sintomatología del vértigo benigno posicional paroxístico y que maniobra utilizas para demostrarlo? • R = Benign paroxysmal positional vertigo (BPPV), which is characterized by sudden-onset brief episodes of vertigo LASTING LESS THAN A MINUTE. The symptoms are USUALLY BROUGHT ON BY HEAD MOVEMENT. The cause is commonly attributed to CALCIUM DEBRIS IN THE SEMICIRCULAR CANALS, KNOWN AS CANALITHIASIS. The debris is loose otoconia (calcium carbonate) within the utricular sac. Although BPPV can occur after head trauma, there is usually no obvious precipitating factor. IT GENERALLY ABATES SPONTANEOUSLY AND CAN BE TREATED WITH VESTIBULAR REHABILITATION. A DIX-HALLPIKE MANEUVER REPRODUCES HIS SYMPTOMS • - A 48-year-old man complains of recurrent episodes of sudden-onset dizziness. He notices an abrupt onset of a spinning sensation when sitting up or lying down in bed. The symptoms last for 30 seconds and then resolve completely. He has no hearing change or other neurologic symptoms, and his physical examination is completely normal. A Dix-Hallpike maneuver reproduces his symptoms. Which of the following is the most likely mechanism for his vertigo symptoms? • (A) basilar migraine • (B) brain stem ischemic events • (C) benign cerebellar tumors • (D) calcium debris (calcium carbonate crystals) in the semicircular canals • (E) Meniere’s disease
  • 46. ARNOLD CHAIRI • Que alteraciones se manifiestan en la malformación de Arnold Chairi? 1) Desplazamiento de las amígdalas cerebelosas del bulbo raquídeo y del 4to ventrículo hacia el conducto medular 2) Se puede diagnosticar desde la vida fetal por medio de us.
  • 47. EVC • Cual es la duración de un ataque isquémico transitorio? • R = Dura < 24 HRS y el DÉFICIT CLÍNICO SE RESUELVE EN MENOS DE 1-2 HRS. Se debe utilizar ASA COMO PROFILAXIS. • Cuanto debe medir un infarto lacunar y como se debe ver en la TAC? • R = Mide <5 MM de diámetro, en la TAC se observan como LESIONES PEQUEÑAS EN SACABOCADO HIPODENSAS, con RESOLUCIÓN parcial o completa de 4-6 SEMANAS • Que sintomatología da el EVC en arteria cerebral anterior? • R = DEBILIDAD y perdida sensitiva en la pierna contralateral y debilidad leve en brazo. • Que cambios produce el infarto cerebral anterior? • R = CAMBIOS CONDUCTUALES y alteración de la MEMORIA. • Que sintomatología da la obstrucción de la arteria cerebral media? • R = AFASIA, HEMIPLEGIA CONTRALATERAL, perdida sensitiva, HEMIANOPSIA HOMÓNIMA, desviación ocular al lado de la lesión.
  • 48.
  • 49. EVC • Que sintomatología causa la obstrucción de la ARTERIA CEREBRAL ½ en su PORCIÓN POSTERIOR? • R = Causa AFASIA DE WERNIKE y HEMIANOPSIA HOMONIMA • - A 74-year-old woman develops acute neurologic symptoms and presents to the emergency room. An urgent magnetic resonance imaging (MRI) scan demonstrates acute occlusion in the right posterior cerebral artery. Which of the following clinical symptoms is she most likely to have? • (A) homonymous hemianopia • (B) total blindness • (C) expressive aphasia • (D) ataxia and dysarthria • (E) a right-sided hemiplegia • -Occlusion of the right posterior cerebral artery is most likely to cause homonymous hemianopia. This artery conveys blood to the inferior and medial portion of the posterior temporal and occipital lobes and to the optic thalamus. • - A 63-year-old man developed a transient episode of vertigo, slurred speech, diplopia, and paresthesias. He is symptom-free now, and clinical examination is entirely normal. His pastmedical history is significant for hypertension and dyslipidemia. Which of the following is the most likely cause for symptoms? • (A) posterior circulation transient ischemic attack (TIA) • (B) anterior communicating artery aneurysm • (C) hypertensive encephalopathy • (D) pseudobulbar palsy • (E) occlusion of the middle cerebral artery • -Posterior circulation TIA is suggested by the transient episodes. The basilar artery is formed by the two vertebral arteries and supplies the pons, the midbrain, and the cerebellum. With vertebrobasilar TIAs, tinnitus, vertigo, diplopia, ataxia, hemiparesis, and bilateral visual impairment are common findings.
  • 50. EVC • Cual es el cuadro clínico de la obstrucción de la arteria basilar? • R = Alteración del estado de alerta, parálisis de los nervios craneales y caídas. • Como manejas la hipertensión maligna? • R = Esta entidad se da con una TAD >130, esta se debe REDUCIR A 1/3 pero NUNCA a un VALOR MENOR DE 95 MM HG. NO se debe usar NEFEDIPINO sublingual. En el caso de HIPERTENSIÓN INTRACRANEAL NO SE DEBE USAR ESTEROIDES. • Como se procede ante un EVC < 3 hrs? • R = TROMBOLISIS IV con activador del plasminogeno no tisular. En los que esta CONTRAINDICADA ESTA TERAPIA son aquellos con: 1) TAS > 185 mm Hg o TAD > 110 2) Sintomatología leve o que mejore de forma rápida 3) CONVULSIONES al inicio de TCE en los 3 MESES PREVIOS 4) CIRUGÍA MAYOR 2 semanas previas 5) Glucemia < 50 o > 400 6) PLAQUETAS < 100,000 7) MUJERES EMBARAZADAS o en lactancia 8) Ingesta actual de anticoagulantes orales con TP > 15 • Cuales son los datos clínicos de una hemorragia intracraneal? • R = Además de depender de la localización del hematoma suele observarse CEFALEA, CRISIS CONVULSIVAS, NAUSEA Y VOMITO.
  • 51. EVC • Cual es el cuadro clínico de la HEMORRAGIA SUBARACNOIDEA? 1) Se debe principalmente a la ROTURA DE UN ANEURISMASECUNDARIA A UN TRAUMATISMO en pacientes con ANTECEDENTE DE HAS 2) CEFALEA SUBITA, intensa y pulsátil espontanea o ASOCIADA A ALGÚN ESFUERZO FISICO. 3) El px lo refiere como EL DOLOR MAS INTENSO QUE HA TENIDO EN SU VIDA 4) SE PUEDE ACOMPAÑAR DE NAUSEA, VOMITO, FONOFOBIA Y FOTOFOBIA. 5) Si la evolución es rápida el px puede evolucionar RAPIDAMENTE A ESTADO DE COMA. 6) Este tipo de hemorragia puede presentarse como MUERTE SUBITA. 7) Para su diagnostico se utiliza la TAC, si hay DUDA DX SE REALIZA PUNCIÓN LUMBAR PARA BILIRRUBINA Y CRENOCITOS QUE CONFIRMAN EL DIAGNOSTICO 8) LA HIPERGLUCEMIA Y EL AUMENTO DE LA TEMPERATURA SE ASOCIAN A MAL PRONÓSTICO. • Cual es el manejo de la HEMORRAGIA SUBARACNOIDEA? 1) REPOSO ABSOLUTO 2) Monitorización 3) Vendaje de miembros inferiores 4) No estimulación 5) ANALGÉSICOS IV 6) NIMODIPINA ORAL para vasoespasmo cerebral sintomático 7) HIPERVOLEMIA PROFILÁCTICA para prevenir vasoespasmo cerebral • Cuales son las características de la HEMORRAGIA SUBDURAL? 1) Son hematomas localizados entre la DURAMADRE Y LA ARACNOIDES 2) TCE CON GLASGOW < 8 3) AGUDO < 24 HR, SUBAGUDO >24 HRS Y < 2 SEMANAS Y CRÓNICO > 2 SEMANAS 4) Los síntomas son FOCALIZACIÓN E HIPERTENSIÓN INTRACRANEANA; en las FORMAS SUBAGUDA Y CRÓNICA PREDOMINAN LA CEFALEA Y DEBILIDAD. 5) Aquellos hematomas que DESPLACEN EL SEPTUM PELUCIDUM > 5 MM de la línea media DEBEN SER EVACUADOS.
  • 52. EVC • Cuales son las características de la HEMORRAGIA EPIDURAL? 1) DISMINUCIÓN DEL ESTADO DE ALERTA seguido de 2) INTERVALO DE LUCIDEZ y posteriormente 3) DETERIORO NEUROLÓGICO PROGRESIVO • Cuales son las características de la trombosis venosa cerebral? 1) 90% presenta como síntoma principal CEFALEA de inicio gradual 2) Debe sospecharse ante un paciente con TAC Y PUNCIÓN LUMBAR NORMAL 3) 40% convulsiones 4) En la TROMBOSIS DEL SENO SAGITAL puede haber DELIRIUM Y AMNESIA. 5) A la exploración física con papiledema y en trombosis venosa focalización 6) El ESTÁNDAR DE ORO PARA EL DIAGNOSTICO ES UNA ANGIOGRAFÍA EN FASE VENOSA ASÍ COMO RM CON VENOGRAFÍA 7) El tratamiento de elección es la ANTI COAGULACIÓN formal por lo menos SEIS MESES debido al riesgo de recurrencia así como de tromboembolia pulmonar.
  • 53.
  • 54.
  • 55.
  • 56.
  • 57.
  • 58.
  • 59.
  • 60. TUMORES SNC • Cuales son las manifestaciones clínicas de una TUMORACIÓN EN EL LÓBULO FRONTAL? • R = Declinación progresiva del intelecto y CAMBIOS EN LA PERSONALIDAD. • Cuales son las manifestaciones clínicas que te da una TUMORACIÓN EN EL LÓBULO TEMPORAL? 1) Si afectan el TERCIO ANTERIOR se presentara EPILEPSIA Y TRASTORNOS DE LA CONDUCTA como agresividad e impulsividad 2) SI afecta los 2/3 POSTERIORES se pueden manifestar con HEMIANOPSIA LATERAL HOMÓNIMA 3) Convulsiones con alucinaciones olfativas o gustativas. Alteraciones en el campo visual. • Cuales son las manifestaciones clínicas que da una TUMORACIÓN EN EL LÓBULO PARIETAL? • R = TRASTORNOS CONTRA LATERALES DE LA SENSIBILIDAD. ESTEREOGNOSIA (no reconocen objetos colocados en la mano), SINDROME DE GERTSMAN “combinación de alexia, agrafia, acalculia y agnosia digital”. • Que manifestaciones clínicas da una TUMORACIÓN EN EL LÓBULO OCCIPITAL? • R = HEMIANOPSIA HOMÓNIMA CRUZADA o defecto del campo visual, PROSOPAGNOSIA ( no reconoce familiares) • Que manifestaciones clínicas da la lesión en el TALLO CEREBRAL Y CEREBELOSAS? • R = PARÁLISIS DE LOS PARES CRANEALES, ATAXIA, incoordinación, nistagmo.
  • 61. TUMORES DEL SNC • Cual es el tipo de tumor mas frecuente en la medula espinal? • R = Ependimoma • Que es la enfermedad de Von Recklinghausen? • R = Es una alteración en el CROMOSOMA 17, llamado NEUROFIBROMATOSIS TIPO I que consiste en maculas MÚLTIPLES HIPERPIGMENTADAS Y NEUROFIBROMAS • - A 10-year-old boy has multiple tan-colored patches on his skin, and freckle-like skin changes in his armpit area. The rest of clinical examination is normal. Which of the following conditions is also found in patients withthis disorder as they get older? • (A) bilateral eighth nerve tumors • (B) irregular small pupils • (C) multiple cutaneous and subcutaneous tumors • (D) cataracts • (E) hip involvement • -The two common forms of neurofibromatosis (NF-1 and NF-2) are genetically distinct. NF-1 is the type with multiple café au lait spots and is associated with axillary or inguinal freckling, iris hamartomas (Lisch nodules), peripheral neurofibromas, and bony abnormalities (including kyphoscoliosis). NF-2 is associated with CNS tumors, particularly bilateral eighth nerve tumors. Skin lesions are spare or absent, and early lens opacities can occur.
  • 62. TUMORES DEL SNC• Que caracteriza a la neurofibromatosis tipo II? • R = Es una alteración en el CROMOSOMA 22. Provoca TUMORES DEL 8vo PAR CRANEAL • Que es el síndrome de STURGE-WEBER? • R = ANGIOMA CAPILAR CUTÁNEO congénito que puede ocasionar ANGIOMA DE COROIDES • Cuales son las características de los astrocitomas (gliomas)? 1) Tumores intracraneales MAS FRECUENTE EN NIÑOS y jóvenes entre 20 y 40ª. 2) EL ASTROCITOMA PILOCITICO es la FORMA MAS BENIGNA de los astrocitomas, PREDOMINA EN NIÑOS y adultos jóvenes, afectando PRINCIPALMENTE AL CEREBELO. • Cuales son las características de los OLIGODENDROGLIOMAS? 1) Estos tumores suelen tener zonas de CLACIFICACIONES Y HEMORRAGIAS 2) Tumor de crecimiento lento • Cuales son las características de los ependimomas? 1) Junto al sistema ventricular REVESTIDO DEL EPÉNDIMO 2) En los niños aparecen principalmente en la primera década de la vida DENTRO DE LOS VENTRÍCULOS. • Cuales son las características de los germinomas? • R = Estos tumores tienen muchas características comunes a las neoplasias de células germinales que se localizan en las gónadas, incluidos los marcadores tumorales • Cual es el cuadro clínico del meduloblastoma? 1) CRISIS CONVULSIVAS PARCIALES O GENERALIZADAS 2) Síndrome de hipertensión intracraneana caracterizado por la triada de: CEFALEA, VOMITO Y EDEMA DE PAPILA BILATERAL. 3) Alteraciones cognitivas que se pueden presentar en tumores del lóbulo frontal.
  • 63.
  • 64. PARKINSON • Que tratamiento resulta efectivo en el temblor esencial benigno? • R = PROPANOLOL y si falla primidona o CLONACEPAM • Que medicamentos causan PARKINSON-LIKE? • R = Neurolépticos, METOCLOPRAMIDA o reseprina. • Cual es el neurotransmisor involucrado en Parkinson y cual es la fisiopatologia? 1) DOPAMINA, habiendo un desequilibro con acetilcolina 2) HISTOLOGICAMENTE la enfermedad de Parkinson se caracteriza por DEGENERACIÓN NEURONAL DE LA SUSTANCIA NIGRA, PARS COMPACTA (PRODUCTORA DE DOPAMINA) y las fibras que la comunica con el estriado. 3) En el microscopio se encuentran los CUERPOS DE LEWY en la sustancia nigra pars compacta (CUERPOS DE INCLUSION INTRANEURONAL, EOSINOFILOS CONSTITUIDOS POR ALTAS CONCENTRACIONES DE LA PROTEINA ALFA-SINUCLEINA). • Cual es el cuadro clínico del Parkinson? 1) Motoras: TEMBLOR EN REPOSO, RIGIDEZ y bradiscinecia. CON LOS MOVIMIENTOS VOLUNTARIOS SE ATENUA EL TEMBLOR Y SE EXACERBA CON LAS EMOCIONES. La bradicinecia se manifiesta como lentitud de movimientos voluntarios. El paciente presenta PERDIDA DE LA EXPRESIÓN FACIAL, menores movimientos de los labios y lengua al hablar (hipofonia), PERDIDA DE LOS MOVIMIENTOS FINOS DE ESCRITURA (micrografía) y manipulación de objetos pequeños, DIFICULTAD PARA LEVANTARSE DE UNA SILLA E INICIAR LA MARCHA la cual se describe como FESTINANTE CON PASOS CORTOS Y RÁPIDOS (APRESURADA). 2) Dermatológica: SEBORREA DE CUERO CABELLUDO y cara con aumento de la sudoración. 3) Psiquiátrica: DEPRESIÓN.
  • 65. PARKINSON • Que es la enfermedad de Wilson en el dx diferencial de Parkinson? • R = Inicio a temprana edad, ANILLOS DE KAYSER-FLEISCHER, hepatitis crónica y aumento de la CONCENTRACIÓN TISULAR DE COBRE. • - A 24-year-old man presents with mild jaundice, tremor, and personality changes. Examination reveals slowness of finger movement, rigidity, and coarse tremor of the outstretched hands. As well there is abnormal slow movement of the tongue and pharynx resulting in a change in speech and occasional difficulty swallowing. He is icteric, the liver span is 10 cm, and no spleen is palpable. Which of the following findings is most likely seen in this condition? • (A) a reduction of copper excretion in the urine • (B) an increase of the serum ceruloplasmin content • (C) no renal involvement • (D) retention of normal neurologic movements • (E) a peculiar greenish-brown pigmentation of the cornea • - In Wilson’s disease, there is usually a reduction of the serum ceruloplasmin content. Signs and symptoms of injury to the basal ganglia are accompanied by cirrhosis of the liver. Renal involvement is characterized by persistent aminoaciduria. The most common neurologic finding is tremor. The corneal pigmentation (Kayser- Fleischer ring) is the most important diagnostic finding on physical examination. If it is absent, any neurologic findings cannot be ascribed to Wilson’s disease. • Cual es el estándar de oro en el diagnostico de Parkinson? 1) La NECROPSIA 2) RM donde se observa REDUCCIÓN DE LA SUSTANCIA NEGRA PARS COMPACTA en la porción mesencefalica superior. 3) PET donde hay MENOR CAPTACIÓN del REACTIVO FLUORO-1-DOPA en el cuerpo estriado.
  • 66. PARKINSON • Cual es el tratamiento del Parkinson? 1) LEVODOPA en pacientes MAYORES A 70ª que es AGONISTA DE LA DOPAMINA utilizada en combinación con CARBIDOPA prevenir la conversión periférica de la levodopa a dopamina por acción de la ENZIMA DOPADESCARBOXILASA. 2) ANTICOLINERGICOS: en pacientes MENORES A 70ª, CONTRAINDICADOS EN HPB Y GLAUCOMA DE ÁNGULO CERRADO. • Que mecanismo de acción tiene la carbidopa? • R = INHIBE LA ENZIMA QUE CONVIERTE LA LEVODOPA A DOPAMINA • Cuales son los agonistas de la dopamina utilizados en Parkinson? • R = BROMOCRIPTINA y pergolida. • Como actúan los inhibidores de la COMT? • R = Inhibidores de la Catecolamina-0-Metiltransferasa REDUCEN EL METABOLISMO DE LA LEVODOPA A 3- METILDOPA y por lo tanto alteran la farmacocinética de la levodopa en plasma llevando a NIVELES MAS SOSTENIDOS DE DOPAMINA PLASMÁTICA: TOLCAPONA/ ENTAGAPONA • Cuales son los fármacos ANTIPSICOTICOS en Parkinson? • R = RISPERIDONA, clozapina: pudiendo esta causar depleción de MO
  • 67. PARKINSON • Que medida quirúrgica se puede utilizar en Parkinson refractario a tx medico? • R = TALAMOTOMIA o PALIDOTOMIA de 1 solo lado. • - Which of the following is the most likely finding in a 79-year-old woman with Parkinson’s disease? • (A) constant fine tremor • (B) muscle atrophy • (C) akinesia • (D) pupillary constriction • (E) spontaneous remission • - The characteristic triad in Parkinson’s disease (Tremor, Rigidity, Akinesia) has been expanded to include Postural instability. This forms the mnemonic TRAP. Autonomic instability is also common. Findings on examination also include masklike facies, dysarthria, stooped posture, and abnormal gait • Que fármacos producen Parkinsonismo? 1) FENOTIACINAS, tioxantenos, butiprofenonas, neurolépticos, valproato y fluoxetina. 2) Produciendo SÍNTOMAS PARECIDOS A LOS DEL PARKINSON causando distonias, rigidez, fascies en mascara, bradicinesia y temblor con menor frecuencia. 3) Los síntomas desaparecen una vez suspendido el fármaco. • Que entidades conforman al Parkinsonismo plus? • R = Son síndromes que incluyen SIGNOS Y SÍNTOMAS DE LA ENFERMEDAD DE PARKINSON RELACIONADOS CON ANORMALIDADES EN PROTEÍNA TAU tales como: 1) Paralisis supranuclear progresiva: Es un trastorno neurodegenerativo que se presenta en la 6ta a 7ma década de la vida y se caracterizan por rigidez, acinesia o bradicinesia, inestabilidad, caídas, disartria, disfagia y demencia. En la RM se observa atrofia del coliculo superior en el mesencéfalo. La respuesta al tratamiento con agonistas de dopamina y levodopa es pobre. 2) Degeneracion corticobasal: Los síntomas se desarrollan en la sexta década de la vida incluyendo rigidez focal o asimétrica, bradicinesia, temblor de acción y reposo y distonía marcada. En la RM se observa perdida cortical localizada en la zona superior de los lóbulos frontales y parietal contralaterales. 3) Enfermedad difusa de los cuerpos de Lewy: Es un TRASTORNO NEURODEGENERATIVO caracterizado por la presencia de síntomas parkinsonianos y alteraciones neuropsiquiatricas comúnmente acompañados por DEMENCIA.
  • 68.
  • 69.
  • 70.
  • 71.
  • 72. HUNTINGTON • En que gen se ha identificado la enfermedad de Huntington? • R = Herencia AD CROMOSOMA 4 • Cual es el cuadro clínico de la enfermedad de Huntington? • R = Aparece entre los 30-50 años. MOVIMIENTOS ANORMALES y CAMBIOS INTELECTUALES CON IRRITABILIDAD, ESTADO DEL ANIMO, TERMINANDO EN DEMENCIA. • - A 38-year-old man presents with involuntary facial grimacing, shrugging of the shoulders, and jerking movements of the limb. His father was similarly affected. There is also a history of mood changes for the past 3 months. On examination, he appears restless with intermittent slow movements of his hands and face. He has difficulty performing rapid finger movements, and tone is decreased in the upper and lower limbs. Which of the following is most likely to represent the progression of his illness? • (A) a normal life span • (B) a 50% chance of only male children being similarly affected • (C) mental deterioration • (D) eventual development of rigidity • (E) development of hemiparesis • - This is a case of Huntington’s chorea. It is an autosomal dominant gene (found on the short arm of chromosome 4), and male and female children are equally affected. Movement disorder, mental deterioration, and personality change are the hallmarks of the disease, but can be very subtle initially. The disease starts typically between ages 35 and 40 (although the variation is wide) and runs its course in about 15 years. The akinetic rigid variety (Westphal variant) of Huntington’s typically has a childhood onset.
  • 73. HUNTINGTON • Que datos de TAC se obtienen en corea de Huntington? • R = Atrofia cerebral y del núcleo caudado • Cuando hay corea sin antecedentes se duda del dx, pero que causa la corea de Syndenham? • R = Es autolimitada por infección por estreptococo del Gpo A • Que padecimiento AD con datos clínicos similares a CH se debe a mutación del gen 12 y con ascendencia japonesa? • R = Atrofia dentorubro palidolisiana con tx igual a CH • Cual es el neurotransmisor involucrado en corea de hungtinton y que tratamiento bloqueador de receptores de ese neurotransmisor controlan la discinecia y cambio conductual? • R = GABA y se utiliza HALOPERIDOL
  • 74. HUNTINGTON • - A 38-year-old man presents with involuntary facial grimacing, shrugging of the shoulders, and jerking movements of the limb. His father was similarly affected. There is also a history of mood changes for the past 3 months. On examination, he appears restless with intermittent slow movements of his hands and face. He has difficulty performing rapid finger movements, and tone is decreased in the upper and lower limbs. Which of the following treatments is helpful in suppressing the movements? • (A) gamma-aminobutyric acid (GABA)- mimetic agents • (B) inhibitors of GABA metabolism • (C) cholinergic agents • (D) dopamine receptor blockers • (E) centrally acting cholinesterase inhibitors • - There are disturbances of norepinephrine, glutamic acid decarboxylase, choline acetyltransferase, GABA, acetylcholine, and somatostatin, but their significance is poorly understood. Dopamine blocking agents (e.g., haloperidol) can be used to treat psychosis and ameliorate chorea but do not alter the course of disease. Presynaptic dopamine depletors such as clozapine, reserpine, or tetrabenazine can be used for chorea as well, but have significant side effects. Antidepressants are helpful for symptomatic treatment. Most patients eventually end up in an institution • - A 44-year-old man presents with involuntary movements of his face, shoulders, and arms. His father had a similar condition. There is also a history of mood changes for the past 6 months. On examination, he appears restless with intermittent slow movements of his hands and face. He has difficulty performing rapid finger movements, and tone is decreased in the upper and lower limbs. Which of the following is the most likely diagnosis? • (A) Parkinson’s disease • (B) Huntington’s chorea • (C) amyotrophic lateral sclerosis (ALS) • (D) spinal muscular atrophy • (E) Sydenham’s chorea • - Huntington’s chorea is the most likely diagnosis given the hereditary nature of this patient’s illness (autosomal dominants). It differs from Sydenham’s chorea by its gradual onset and slow choreic movements versus brusque jerks seen in Sydenham’s. The caudate nucleus and putamen are both severely involved in Huntington’s chorea, and degeneration of the caudate nucleus results in enlarged lateral ventricles (with a “butterfly” appearance on CT). Atrophy is very widespread in the brain and includes the cerebral cortex. Adecrease in glucose metabolism as revealed on positron emission tomography (PET) scan precedes the evidence of tissue loss.
  • 75. ALZHEIMER • Cual es la manifestación principal de la enfermedad de Alzheimer? • Déficit de ACETILCOLINESTERASA • La PERDIDA DE MEMORIA a corto plazo • Cual es la manifestación encontrada en TAC y Biopsia en Alzheimer? 1) Alzheimer’s disease can be quite diffuse, but there is particular INVOLVEMENT OF THE MEDIAL TEMPORAL LOBES AND CORTICAL ASSOCIATION AREAS. 2) THE ATROPHY OF THE HIPPOCAMPUS IS PARTICULARLY MARKED. 3) Microscopic examination reveals NEUROFIBRILLARY TANGLES AND AMYLOID PLAQUES. • - A 69-year-old woman presents to the clinic with memory difficulty. The patient’s daughter is concerned because she is having difficulty doing her finances, such as paying bills. Memory impairment testing reveals the poor ability to generate lists of words or copy diagrams (intersecting pentagons). Her remaining physical examination is normal. Which of the following anatomic findings is most likely with her condition? • (A) atrophy of the medial temporal lobes • (B) atrophy of the entire frontal and temporal lobes • (C) cranial nerve involvement • (D) transient episodes of hemiplegia • (E) atrophy of the caudate
  • 76.
  • 77.
  • 78.
  • 79.
  • 80.
  • 82.
  • 83. DISTONIA DE TORSIÓN IDEOPATICO • En que consiste la distonía de torsión idiopático? • R = En movimientos y posturas diatónicas con antecedentes de nacimiento y desarrollo normales que inician en la infancia y mas tarde son permanentes • Cual es el cuadro clínico de la distonía de torsión idiopático? • R = Movimientos y posturas anormales en paciente con ANTECEDENTES DE NACIMIENTOS NORMALES. En la infancia comúnmente inicia en piernas. • Como diagnosticas distonía de torsión idiopático? 1) Exclusión de hipoxia neonatal, Kernicterus, o trauma al nacimiento. 2) Las investigaciones con TAC incluida no revelan alteraciones neurológicas • Cual es el manejo de la distonía de torsión idiopático? • R = NO HAY, SE PUEDE ADMINISTRAR LEVODOPA.
  • 84. DISTONIA DE TORSIÓN FOCAL • Que caracteriza a la distonía de torsión focal? • R = Manifestaciones de la distonía de torsión idiopático pero focal que puede presentarse en la infancia por herencia o sin ella en adulto • Cuales son las manifestaciones clínicas mas frecuentes de la distonía de torsión focal? 1) Blefaroespasmo y distonia bucomandibular. 2) Torticolitis espasmódica. • Que tratamiento resulta útil pero no curativo en la distonia de torsión focal? • R = Inyección de toxina botulínica
  • 85. GILLES DE LA TOURETTE • Que caracteriza al síndrome de Gilles de la Tourette? • R = Los TICS MOTORES constituyen el 80% siendo común >21ª afectando PRINCIPALMENTE LA CARA. • Cual es el cuadro clínico del síndrome de Gilles de la Tourette? 1) Tics motores: Olfateo, PARPADEO, fruncir el ceno, ENCOGER LOS HOMBROS O ASENTIR CON LA CABEZA. 2) Tics fónicos: Murmullos, quejidos, ladridos y aullidos, coprolalia. • Cual es el tratamiento de elección del sx de Gilles de la Tourette? • R = HALOPERIDOL
  • 86. ESCLEROSIS MULTIPLE • Que caracteriza a la esclerosis múltiple? 1) Quizá tenga base autoinmune con desmielinizacion con gliosis reactiva en sustancia blanca encefálica y de la medula espinal 2) Se le ha asociado con Chlamydia pneumoniae, virus herpes 6 y adenovirus 3) El gen HLA-DRB1 en el cromosoma 6p21 es, entre los muchos genes HLA asociados con esclerosis múltiple. 4) El OLIGODENDROCITO encargado de la mielinizacion ES EL BLANCO PRINCIPAL del proceso inflamatorio principalmente mediado por LT CD 8 CITO TÓXICOS y macrófagos. 5) Patológicamente se distingue por placas de desmielinizacion afectando mas frecuentemente la sustancia blanca periventricular • Cual es la clasificación de la esclerosis múltiple? 1) Existen 4 tipos básicos de esclerosis múltiples según su progresión en brote - remisión, primaria progresiva, secundaria progresiva y progresiva-recurrente. 2) 90% de los pacientes comienza con el TIPO BROTE- REMISIÓN caracterizado por la presentación de uno a dos ataques por year con remisiones y recuperaciones en los meses posteriores. • Cual es el cuadro clínico de la esclerosis múltiple? 1) La PRESENTACIÓN COMÚN ES DEBILIDAD, ENTUMECIMIENTO, HORMIGUEO, INESTABILIDAD DE UNA EXTREMIDAD, paraparesia espástica, NEURITIS RETROBULBAR, trastornos esfinterianos. Los síntomas desaparecen de días a semanas. En el examen aparecen ATROFIA ÓPTICA, nistagmo, disartria y déficit sensitivo en extremidades. 2) Patrón ataque - recaída - exacerbación, definiéndose ataque como el cuadro agudo de disfunción del SNC que dure mas de 24 hrs en ausencia de fiebre, infecciones o alteraciones metabólicas. 3) FENÓMENO DE UTHOFF, el cual consiste en la aparición o exacerbación de los síntomas tras el ejercicio o un baño con agua caliente 4) TANTO LAS TAREAS COGNITIVAS COMO LA ACTIVIDAD FÍSICA PROVOCAN CANSANCIO PROFUNDO Y LOS PACIENTES TARDAN MAS TIEMPO EN RECUPERARSE 5) El CANSANCIO es uno de los síntomas mas comunes y discapacitantes de la enfermedad. 6) La afectación de la vía visual es casi una regla con la presencia de NEURITIS ÓPTICA que se manifiesta como CEGUERA MONO O BINOCULAR. 7) EL SIGNO DE LHERMITTE (sensación de electricidad que va de la medula hacia las extremidades al flexionar el cuello).
  • 87. ESCLEROSIS MULTIPLE • -A 22-year-old woman presents with acute vision loss and pain in the left eye, but no other symptoms. On examination she appears well, visual acuity is not possible, and she can only perceive movement and bright light. The direct papillary reflex is absent but the indirect (consensual) response is normal. The optic disc is edematous. Which of the following symptoms is also most likely present in patients with this condition? • (A) limb weakness • (B) hemiplegia • (C) cervical myelopathy • (D) sphincter impairment • (E) seizures • - This patient has multiple sclerosis (MS). Weakness or numbness in one or more limbs is the initial manifestation of disease in about half the patients. Other common initial presentations include optic neuritis (25%) and acute myelitis. Hemiplegia, seizures, and cervical myelopathy (in older patients) occur occasionally as the initial manifestation. Sphincter impairment usually occurs later in the disease. • -A 27-year-old woman presents with acute vision loss and pain in the left eye, but no other symptoms. On examination, she appears well, visual acuity is not possible, and she can only perceive movement and bright light. The direct pupillary reflex is absent but the indirect (consensual) response is normal. The optic disc is edematous. Which of the following is the most likely diagnosis? • (A) diabetic microvascular disease • (B) arteriosclerosis • (C) trauma • (D) multiple sclerosis • (E) Creutzfeldt-Jakob disease • -Visual loss in multiple sclerosis varies from slight blurring to no light perception. Other eye symptoms include diplopia and pain. The classic syndrome of optic or retrobulbar neuritis occurs commonly at some point in the disease, and it is the presenting symptom in 25% of cases.
  • 88.
  • 89. ESCLEROSIS MULTIPLE • Cual es el estudio de elección para dx de esclerosis múltiple? 1) IRM de ENCÉFALO o MEDULA ESPINAL que demuestra MIELOPATIA FOCAL O MÚLTIPLE EN LA SUSTANCIA BLANCA. Se aplican los criterios de Barkhof y Tintore. 2) El estudio del LCR permite valorar la inflamación intratecal. SE BUSCAN BANDAS OLIGOCLONALES DE IgG que no están presentes en suero. Un total de PROTEÍNAS MENOR A 100 Y LEUCOCITOS MENORES A 50 ES LO COMÚN 3) Los POTENCIALES EVOCADOS VISUALES son los mas SENSIBLES Y ESPECÍFICOS, los cuales DENOTAN DESMIELINIZACION. • En que consiste la malformación de Arnold Chari, la cual debe descartarse en esclerosis múltiple? • R = Observándose la región del agujero magno UNA PARTE DEL CEREBELO Y PARTE INFERIOR DEL TALLO ENCEFÁLICO SE DESPLAZAN HACIA EL CONDUCTO CERVICAL produciendo déficit piramidal y cerebeloso en extremidades. • Cual es el tratamiento de esclerosis múltiple? • R = INTERFERON B disminuye exacerbaciones al igual que inmunoglobulina IV. NO HAY FORMA DE DETENER EL AVANCE, se utiliza PREDNISONA EN RECIDIVAS. • Cual es el pronostico de la esclerosis múltiple? • R = Mientras mayor sea el numero y volumen de las lesiones peor será el pronostico.
  • 90. ESCLEROSIS MULTIPLE • En que consiste el fenómeno de Utoff en esclerosis múltiple? • R = EMPEORAMIENTO DE LA VISIÓN al exponer a una persona a TEMPERATURAS ELEVADAS O QUE CURSE CON TEMPERATURA ELEVADA.
  • 91.
  • 92.
  • 93.
  • 96.
  • 97.
  • 98. ATAXIA DE FRIEDREICH • Que es la ataxia de Friedreich? 1) AR, en CROMOSOMA 9 q13-9 q21. 2) Es una MARCHA ATÁXICA, manos torpes, respuesta extensora plantar y disminución de ROTS. 3) The pathologic changes are found in the SPINAL CORD TRACTS. Degeneration is seen in the POSTERIOR COLUMNS, the lateral corticospinal tract, and the spinocerebellar tracts. 4) PIE CAVO BILATERAL • -A 19-year-old man has had progressive ataxia of gait and great difficulty in running. In the past year, he has developed hand clumsiness. Physical examination reveals pes cavus, kyphoscoliosis, and both cerebellar and sensory changes in the legs. There is a positive family history of Friedreich’s ataxia. Where are the pathologic changes seen in this condition most likely to be found? • (A) spinal cord tracts • (B) basal ganglia • (C) cerebral cortex • (D) peripheral autonomic nerves • (E) peripheral motor nerves • - This young man has Friedreich’s ataxia, associated with a gene defect on chromosome 9. The pathologic changes are found in the spinal cord tracts. Degeneration is seen in the posterior columns, the lateral corticospinal tract, and the spinocerebellar tracts. Ataxia, sensory loss, nystagmus, reflex changes, clubfeet, and kyphoscoliosis are the characteristic findings. The heart is frequently involved, and cardiac disease is a common cause of death.
  • 99. DEFICIT VITAMINA E y B12 • Que produce el déficit de vitamina E a nivel SNC? • R = Se presenta degeneración medulocerebelosa que afecta principalmente al cordón posterior de la medula espinal. • Cuales son los datos clínicos de déficit de vitamina E? • R = Ataxia de extremidades, disminución sensitiva, disminución de ROTS, confusión y degeneración pigmentaria de la retina. • Cual es el tratamiento para el déficit de vitamina E? • R = Alfa tocoferilo • Que trastorno en el SNC te ocasiona el déficit de vitamina B12? • Degeneración combinada subaguda de medula espinal. • Se acompaña de polineuropatía, cambios mentales o neuropatía óptica.
  • 100. ENCEFALOPATÍA DE WERNICKE • Que caracteriza la encefalopatía de Wernicke? • R = Déficit de “TIAMINA B1” y es COMÚN EN BORRACHOS. • Cual es el cc de la encefalopatía de Wernike? • R = CONFUSIÓN, ATAXIA Y NISTAGMO que da lugar a oftalmoplejia (parálisis de la mirada conjugada, debilidad del musculo recto externo) • -A 43-year-old man is referred from the emergency department with memory loss and difficulty walking. He was brought in by his wife who has noticed personality changes, truancy from work, and lack of personal care over the past 2 years. On examination he appears unkempt, smells of urine, and is uncooperative. He cannot recall the date or season, and gets angry when asked questions. His answers are often fabricated when checked with his wife. His gait is wide-based, and there is loss of sensation in his feet up to the shins. His motor strength and reflexes are normal. His ocular movements are normal and there is no nystagmus. In the past he has had multiple admissions for alcohol withdrawal. Which of the following is the most likely diagnosis? • (A) Wernicke’s encephalopathy • (B) Wernicke-Korsakoff syndrome • (C) Alzheimer’s dementia • (D) Charcot-Marie-Tooth disease • (E) vascular dementia • -The combination of symptoms is typical of chronic alcohol abuse. The mental symptoms are suggestive of Wernicke-Korsakoff syndrome. Adistal limb sensory-motor neuropathy is also typical of alcoholism. Confusion, tremulousness, and disorientation are typical for acute alcohol intoxication. Wernicke’s encephalopathy is a symptom complex of ophthalmoplegia, ataxia, nystagmus, and acute confusional state
  • 101. DEGENERACIÓN COMBINADA SUBAGUDA • Cual es la etiología de la degeneración combinada subaguda y cual es el cuadro clínico característico? 1) Es causado por la deficiencia de COBALAMINA (VB12) 2) En un inicio encontramos signos y síntomas de lesión de la COLUMNA POSTERIOR (PARESTESIAS DE MANOS Y PIES, INESTABILIDAD DE LA BIPEDESTACIÓN Y MARCHA, ALTERACIONES DE VIBRACIÓN Y POSICIÓN) 3) Después de un tiempo a las 2 semanas posteriores se identifica una paraparesia atáxica simétrica con hiperreflexia o hiporreflexia tendinosa y signo de Babinski.
  • 104. ESLEROSIS LATERAL AMINOTROFICA • Que es la esclerosis lateral aminotrófica? 1) Se refiere a una condición neurología de INICIO EN LA VIDA ADULTA caracterizada por una DEGENERACIÓN PROGRESIVA DE LAS NEURONAS MOTORAS SUPERIORES E INFERIORES. 2) La degeneración de las neuronas de la corteza motora se manifiesta como hiperreflexia, signos positivos de Hoffman, Babinski y clonus. 3) El termino esclerosis lateral se refiere a la DUREZA CUANDO SE PALPAN LAS COLUMNAS LATERALES DE LA MEDULA ESPINAL en las autopsias de pacientes con esta enfermedad. 4) El termino aminotrófica se refiere a la ATROFIA MUSCULAR, DEBILIDAD Y FASCICULACION debidas a la degeneración de las neuronas motoras inferiores. • Que hallazgos muestra la electromiografía al respecto de la esclerosis lateral aminotrófica? • R = Demuestra DENERVACIÓN de POR LO MENOS 3 EXTREMIDADES. • Cual es el manejo de la esclerosis lateral aminotrófica? • R = RILUZOLE QUE ES UN ANTAGONISTA DEL GLUTAMATO Y LA VITAMINA E. Deben ser tratados como si tuvieran lesión de la columna cervical hasta que se demuestre lo contrario.
  • 105. CHARCOT-MARIE-TOOTH • Cuales son las características de la mononeuropatia hereditaria de Charcot-Marie-Tooth? 1) Carácter AD en CROMOSOMA 17. 2) Clínicamente con DEFORMIDADES DEL PIE, TRASTORNOS DE LA MARCHA. 3) El electrodiagnóstico muestra DISMINUCIÓN DE LA CONDUCCIÓN MOTORA Y SENSITIVA (neuropatía motora y sensitiva hereditaria tipo I) NMSH tipo I. 4) NMSH tipo II disminuye solo un poco. 5) Histológicamente hay PERDIDA AXONAL.
  • 106. DEJEWINE-SOTTAS NMSH TIPO III • Que caracteriza a la mononeuropatía hereditaria Dejewine-Sottas NMSH tipo III? 1) AR, polineuropatía progresiva motora y sensitiva con debilidad, ataxia, disminución sensitiva e hiporreflexia. 2) Histopatológicamente con NERVIOS PERIFÉRICOS SENSIBLES Y PALPABLES, desmielinizacion segmentaria, HIPERPLASIA DE CÉLULAS DE SCHWAN.
  • 107. ENFERMEDAD DE REFSUM NMSH TIPO IV • Que caracteriza a la mononeuropatia hereditaria de la enfermedad de Refsum NMSH tipo IV? 1) AR, trastorno en el metabolismo del ACIDO FITANICO. 2) Hay DEGENERACIÓN RETINIANA PIGMENTARIA + polineuropatia sensitivomotora. 3) DISFUNCIÓN AUDITIVA, MIOCARDIOPATÍA. 4) El tratamiento es LA DISMINUCIÓN DEL ACIDO TIFANICO.
  • 108. PORFIRIA • En la porfiria aguda con manifestaciones en los parámetros motores y debilidad proximal, que medicamento resulta útil? 1) HEMATINA. 2) Administración alta de CHBTS, glucosa IV. 3) Propanolol que controla la taquicardia e hipertensión.
  • 109. LYME • Cuales son las manifestaciones neurológicas de la enfermedad de Lyme? 1) Meningitis, 2) Poliradiculopatia, 3) Mononeuropatia multiple y 4) Neuropatía cutánea.
  • 110. SARCOIDOSIS • Cuales son las manifestaciones neurológicas de la sarcoidosis? • R = Parálisis de los pares craneales, en especial el 7 que responde a esteroides
  • 111. SÍNDROME DE GUILLAIN-BARRE • Cual es la etiología del síndrome de Guillain-Barre? 1) Suele asociarse a una infección de vías respiratorias o gastrointestinal. 2) Principalmente el CAMPYLOBACTER JEJUNI Y CMV. • Cuales son los signos y síntomas de la poliradiculoneuropatia progresiva aguda/sub aguda conocido como síndrome de Guillain- Barre? • R = Debilidad ascendente de comienzo en piernas. • Cuales son los estudios para el diagnostico de síndrome de Guillain- Barre? 1) LCR con AUMENTO DE PROTEÍNAS 2) Células normales 3) Electrofisiología con alteraciones 4) Histología que muestra DESMIELINIZACION PRIMARIA • Cual es el manejo del síndrome de Guillain-Barre? • R = Administración de INMUNOGLOBULINA IV O PLASMAFERESIS.
  • 112. MIASTENIA GRAVE • Cual es el cuadro clínico de la miastenia grave? 1) DIPLOPÍA, debilidad de músculos oculares, debilidad progresiva pudiendo causar parálisis respiratoria, PTOSIS. 2) El cuadro clínico confirma DEBILIDAD Y FATIGA DE MÚSCULOS EL CUAL MEJORA CON EL REPOSO. • Como se realiza el diagnostico de miastenia grave? 1) Respuesta a ANTICOLINESTERASA de acción corta con EDROFONIO con vida media de 1 minuto en el cual los PACIENTES CON RESPUESTA TIENEN MEJORÍA NOTORIA DURANTE 5 MINUTOS. 2) NEOSTIGMINA, la respuesta dura 2 hrs, con sulfato de atropina en caso de efectos muscariniocos adversos. 3) ELECTROFISIOLOGIA que muestra DISMINUCIÓN DE LA TRANSMISIÓN. 4) LAB con Ac VS RECEPTOR DE ACETILCOLINA • Cual es el tratamiento de miastenia grave? • R = NEOSTIGMINA. Timectomia en caso de timoma. PROSCRITOS AMINOGLUCOSIDOS por que causan bloqueo neuromuscular
  • 113. MIASTENIA GRAVE • - A 30-year-old woman complains of double vision, and easy fatigue with exercise. The fatigue improves with resting, but it is interfering with her work. Examination reveals ptosis and impaired eye movements with normal pupillary response. The double vision is brought out by asking her to look at the ceiling, and after a sustained interval, the eyes slowly drift down. Which of the following is the most likely diagnosis? • (A) optic atrophy • (B) ophthalmic zoster • (C) paralysis agitans • (D) Horner syndrome • (E) myasthenia gravis • - In myasthenia gravis, weakness of the facial and levator palpebrae muscles produces a characteristic expressionless face, with drooping of the eyelids. Weakness of the ocular muscles may cause paralysis or weakness of individual muscles, paralysis of conjugate gaze, ophthalmoplegia, or a pattern similar to internuclear ophthalmoplegia. The presence of normal pupillary responses to light and accommodation with weakness of extraocular muscles, levators, and orbicularis oculi is almost completely diagnostic of myasthenia
  • 114. SÍNDROME DE LAMBERT-EATON O MIASTENIFORME • Que puede ocasionar el síndrome de lambert-eaton o miasteniforme? 1) En este hay liberación DEFECTUOSA DE ACETILCOLINA. 2) Se presenta con DEBILIDAD DE MÚSCULOS PROXIMALES y se diferencia de miastenia gravis por que LA FUERZA SE AUMENTA CON CONTRACCIÓN SOSTENIDO. • Como diagnosticas el síndrome de lambert-eaton? • R = Electrofisiología por AUMENTO DE LA RESPUESTA MUSCULAR A LA ESTIMULACIÓN • Cual es el manejo del síndrome de lambert-eaton? • R = PLASMAFERESIS, inmunodepresores (azatropina, prednisona).
  • 115. NEUROINFECCION• Ayuda la toma de LCR en el caso de absceso cerebral? • R = NO, de echo ESTA PROSCRITO por que puede causar herniación. • Cual es la complicación mas frecuente del SIDA en el SNC? • R = This is one of the most common neurologic complications of AIDS. Its pathologic substrate is degeneration of the spinal tracts in the posterior and lateral columns, which have a vacuolated microscopic appearance. Although the morphologic changes and clinical manifestations are similar to those associated with vitamin B12 deficiency, the pathogenetic mechanism is probably not related to dietary deficiencies. Since there is no specific clinical or laboratory test available for the diagnosis of this syndrome, VACUOLAR MYELOPATHY IN AIDS PATIENTS REMAINS A DIAGNOSIS OF EXCLUSION. This implies that other HIV-related neurologic complications must be ruled out • -A 56-year-old man is brought to the emergency department by his wife because of memory loss and difficulty walking. She has noticed personality changes, truancy from work, and lack of personal care over the past 1 year. On examination he appears unkempt, smells of urine, and is uncooperative. He cannot recall the date or season,and gets angry when asked questions. His answers are often fabricatedwhen checked with his wife. The blood pressure is 150/90 mm Hg, pulse 100/min, and he is diaphoretic and tremulous. His gait is wide based, and motor strength and reflexes are normal. His ocular movements are normal but there is nystagmus on lateral gaze. In the past he has had multiple admissions for alcohol withdrawal. Which of the following is the most appropriate next step in management? • (A) prophylactic phenytoin administration • (B) prophylactic diazepam administration • (C) prophylactic carbamazepine administration • (D) calcium administration • (E) steroid administration • - Prophylactic administration of diazepam in a withdrawing alcoholic can prevent or reduce severe syndromes such as delirium tremens (DTs). Prophylactic phenytoin, however, is not helpful. Acalm, quiet environment with close observation and frequent reassurance is very important. Vitamin administration (especially thiamine) is important, but frequently, severe magnesium depletion slows improvement.
  • 116. LCR
  • 117. BOTULISMO • Cual es la fisiopatología del botulismo? • R = La toxina EVITA LA LIBERACIÓN DE ACETILCOLINA en las uniones neuromusculares y sinapsis autónomas, transmitida por COMIDAS ENVASADAS • Cual es el cuadro clínico del botulismo? 1) Desarrollo SÚBITO DE DEBILIDAD INTENSA DESPUÉS DE 72 HRS DE LA COMIDA. 2) El típico cuadro es de DIPLOPÍA, VISIÓN BORROSA, midriasis no reactiva, estreñimiento por íleo paralitico, PTOSIS, debilidad facial, DIFICULTAD RESPIRATORIA Y DEBILIDAD EN EXTREMIDADES INFERIORES CON SENSIBILIDAD CONSERVADA. • Como se dx el botulismo? • R = SUERO CON TOXINA. Electrofisiológico con estimulación repetida con aumento de la respuesta muscular de manera progresiva • Cual es el manejo del botulismo? 1) ANTITOXINA en pacientes NO ALÉRGICOS AL SUERO DE CABALLO. 2) GUANIDINA facilita la liberación de acetilcolina
  • 118. MIGRANA • Cual es el cuadro clínico de la migraña? 1) Cefalea intensa, pulsátil, unilateral en la región frontal o temporal. 2) La migraña con aura consiste en un inicio con alteraciones visuales con luces-centilleos o luces de colores, palidez, vértigo, alteraciones sensitivas o debilidad unilateral, 3) Afasia transitoria o alteraciones en el lenguaje que aparecen previamente al establecimiento de la migraña. • A que se refiere el termino estatus migranoso? • R = Se refiere a pacientes ya conocidos con migraña con manifestaciones clínicas por mas de 72 Hrs • Cual es el manejo de la migraña? • R = Triptanos y derivados de la ergotamina
  • 119. MIGRANA • Cuales son las 2 variantes de migraña a considerar? 1) Los síntomas prodrómicos son el resultado de una disfunción en el territorio de la circulación cerebral posterior con sintomatología visual bilateral, ataxia, disartria, vértigo, parestesias de las extremidades e incluso debilidad. Puede haber perdida de la conciencia antes de comenzar la cefalea. 2) Migraña hemipléjica: Es poco frecuente e incluye hemiplejia, que puede persistir días después de calmarse la cefalea. • - An 18-year-old woman has periodic episodes that begin with severely decreased vision, followed by ataxia, dysarthria, and tinnitus. The symptoms last for 30 minutes and are then followed by a throbbing occipital headache. Which of the following is the most likely diagnosis? • (A) vertebral-basilar insufficiency • (B) chronic basilar artery dissection • (C) classic migraine • (D) ophthalmoplegic migraine • (E) basilar migraine • Basilar migraine can be very dramatic, and can resemble ischemia in the territory of the basilar posterior cerebral arteries. The visual symptoms of basilar migraine typically affect the whole of both visual fields, and can even cause temporary cortical blindness. There can also be an alarming period of coma or quadriplegia.
  • 120.
  • 121. CEFALEA TENSIONAL • Cual es el cuadro clínico de la cefalea tensional? 1) Dolor difuso, en banda, de carácter sordo, que puede acompañarse de hiperestesia en el cuero cabelludo y agravarse con el ruido y la luz. 2) Típicamente la distribución del dolor es en forma de banda, bilateral y se extiende desde la frente hacia los temporales y nuca. 3) Como profilaxis pueden utilizarse antidepresivos triciclicos y la amitriptilina es la más eficaz.
  • 122. CEFALEA EN RACIMOS O ENFERMEDAD DE HORTON • Que caracteriza a la cefalea en racimos o enfermedad de Horton? 1) Intenso dolor unilateral periorbitario acompañado de inyección conjuntival, lagrimeo, rinorrea y síndrome de Horner 2) El comienzo del dolor suele ser en la madrugada o al despertar muy temprano. 3) Para su tratamiento son útiles los fármacos utilizados en migraña. 4) Nifedipino y berapamil como parte del manejo profiláctico.
  • 123. NARCOPLEPSIA• Cual es la sintomatología en caso de narcolepsia? • R = The early age of onset and otherwise good health suggest a diagnosis of narcolepsy, which is usually accompanied by other symptomatology. Hypnagogic hallucinations are almost always visual. They occur most frequently at the onset of sleep, either during the day or at night. They are generally very vivid. Cataplexy is a brief loss of muscle power without loss of consciousness. The patient is fully aware of what is going on. The paralysis may be complete or partial. • -A 31-year-old woman complains of excessive sleepiness during the daytime for years despite adequate nighttime sleep. She has episodes of intense drowsiness three to four times a day, even when at work or while eating meals. She has sought medical attention in the past, after falling asleep while driving. She is slender and otherwise healthy and on no medications. Which of the following treatments is most likely indicated for her condition? • (A) a device providing continuous positive airway pressure (CPAP) at night • (B) oral surgery • (C) tracheostomy • (D) amphetamines • (E) benzodiazepines at bedtime • -This woman does not have risk factors for sleep apnea (older age, snoring, obesity) and likely has narcolepsy. Adrenergic stimulant drugs such as methylphenidate or amphetamines help the sleepiness, and tricyclic compounds can help the cataplexy. Strategically planned naps can also be helpful. • -A 25-year-old man complains of excessive sleepiness during the daytime for years despite adequate nighttime sleep. He has sought medical attention after falling asleep while driving. He is slender and otherwise healthy and on no medications. Which of the following symptoms might he also complain about? • (A) excessive snoring (wife’s report) • (B) automatic behavior (wife’s report) • (C) restless sleep (wife’s report) • (D) paresthesias • (E) morning headache • -Automatic behavior with amnesia is a common manifestation of the narcolepsycataplexy syndromes, occurring in 50% of cases. Automatic behavior can be confused with complex partial seizures. Paresthesias are not part of narcolepsy syndrome. Snoring, restless sleep, and morning headache suggest sleep apnea.
  • 125. DISTROFIA MIOTONICA • Cual es la etiología de la distrofia miotonica? • R = Trastorno AD lentamente progresivo entre los 30-40, en ocasiones durante la infancia, localizado en el cromosoma 19. • Cual es el cuadro clínico de la distrofia miotonica? 1) Molestias de rigidez muscular con DISMINUCIÓN DE LA RELAJACIÓN DESPUÉS DE LA CONTRACCIÓN. 2) Hay debilidad y emaciación de músculos faciales, esternocleidomastoideos y distales. 3) Cataratas, calvicie, atrofia testicular y DM • Como dx distrofia miotonica? • R = Electromiografía revela descargas miotonicas adicionales. • Cual es el manejo de la distrofia miotonica? • R = Sulfato de quinidina, procainamida o fenitoina.
  • 126. MIELOPATIA POR VIRUS DE LA LEUCEMIA DE LINFOCITOS T HUMANOS • Como se transmite la mielopatia por virus de la leucemia de linfocitos T humanos? • R = El virus de la leucemia de linfocitos T humanos HTLV-1 se transmite por medio del seno materno, sexual, sangre y agujas contaminadas. • Cuales son los datos de laboratorio que orientan a mielopatia por virus de la leucemia de linfocitos T humanos? • R = Suero y LCR hay AC vs HTLV-1, datos similares a esclerosis multiple en LCR, IRM o EEG. NO HAY TX ESPECIFICO.
  • 127. MIOTONIA CONGENITA • Cual es la etiología de la miotonia congénita? 1) Se hereda de forma AD en cromosoma 19. 2) Se caracteriza por miotonia generalizada sin debilidad en movimiento, hay rigidez muscular que aumenta con el frio y disminuye con el ejercicio. 3) Hipertrofia muscular. • Cual es el manejo de la miotonia congénita? • R = Sulfato de quinidina, procainamida
  • 128. MIOSITIS DE CUERPOS DE INCLUSIÓN • Cual es la etiología de la miositis de cuerpos de inclusión? 1) Origen desconocido. 2) Cuadro clínico comienza arriba de los 20ª con debilidad proximal progresiva, 1ero de extremidades inferiores y después de extremidades superiores. • Cual es el tratamiento de la miositis de cuerpos de inclusión? • R = Inmunoglobulina IV, los esteroides son ineficaces
  • 129. MIOPATÍAS MITOCONDRIALES • Cuales son las características de las miopatías mitocondriales? • R = En la biopsia con tinción modificada de Gomon, se observan fibras rojas que contienen un cumulo de mitocondias anormales. • Cual es el cuadro clínico de las miopatías mitocondriales? 1) Oftalmoplejia progresiva, debilidad de las extremidades exacerbada por actividad. 2) Otros con epilepsia mioclonica o combinación de miopatía encefalopatía, acidosis láctica. 3) Episodios similares al EVC
  • 130. SÍNDROME DE PARALISIS PERIODICA • Cual es la etiología del síndrome de parálisis periódica? • R = Herencia AD, gen 17 en canal de Na. • Cual es el cuadro clínico del síndrome de paralisis periodica? 1) Episodios de debilidad o parálisis flácida acompañándose ocasionalmente de hipokalemia. 2) Suele presentarse al despertar, después del ejercicio. • Cual es el manejo del síndrome de parálisis periódica? • R = Dieta baja en CHBTS, acetazolamida, Cloruro de K VO o IV. Glucosa e insulina.
  • 131. Consejos 1. Leer diariamente y en bloques 2. Adiós Partys un tiempo 3. Has ejercicio y come bien durante el estudio 4. Toma algún curso bueno si tienes la posibilidad 5. Ten Fe.
  • 132. BIBLIOGRAFIA • EXARMED • PAPADAKIS • CTO • HARRISON • AMIR • USMLE STEPS